CINXE.COM
undergraduate education - Inquiry about my note-taking skill - Mathematics Educators Stack Exchange
<!DOCTYPE html> <html itemscope itemtype="https://schema.org/QAPage" class="html__responsive " lang="en"> <head> <title>undergraduate education - Inquiry about my note-taking skill - Mathematics Educators Stack Exchange</title> <link rel="shortcut icon" href="https://cdn.sstatic.net/Sites/matheducators/Img/favicon.ico?v=20217095b863"> <link rel="apple-touch-icon" href="https://cdn.sstatic.net/Sites/matheducators/Img/apple-touch-icon.png?v=41c548fc9c75"> <link rel="image_src" href="https://cdn.sstatic.net/Sites/matheducators/Img/apple-touch-icon.png?v=41c548fc9c75"> <link rel="search" type="application/opensearchdescription+xml" title="Mathematics Educators Stack Exchange" href="/opensearch.xml"> <link rel="canonical" href="https://matheducators.stackexchange.com/questions/8456/inquiry-about-my-note-taking-skill" /> <meta name="viewport" content="width=device-width, height=device-height, initial-scale=1.0, minimum-scale=1.0"> <meta name="bingbot" content="noarchive"> <meta property="og:type" content= "website" /> <meta property="og:url" content="https://matheducators.stackexchange.com/questions/8456/inquiry-about-my-note-taking-skill"/> <meta property="og:site_name" content="Mathematics Educators Stack Exchange" /> <meta property="og:image" itemprop="image primaryImageOfPage" content="https://cdn.sstatic.net/Sites/matheducators/Img/apple-touch-icon@2.png?v=adf129f0fc54" /> <meta name="twitter:card" content="summary"/> <meta name="twitter:domain" content="matheducators.stackexchange.com"/> <meta name="twitter:title" property="og:title" itemprop="name" content="Inquiry about my note-taking skill" /> <meta name="twitter:description" property="og:description" itemprop="description" content="I am a rising college junior in US with a major in mathematics. I recently noticed a problem in my note-taking skill in the mathematics, both from the textbooks and lectures. When I was a microbi..." /> <script id="webpack-public-path" type="text/uri-list">https://cdn.sstatic.net/</script> <script src="https://ajax.googleapis.com/ajax/libs/jquery/3.7.1/jquery.min.js"></script> <script defer src="https://cdn.sstatic.net/Js/third-party/npm/@stackoverflow/stacks/dist/js/stacks.min.js?v=fe3ef2b1305f"></script> <script src="https://cdn.sstatic.net/Js/stub.en.js?v=44cbb4d4d062"></script> <link rel="stylesheet" type="text/css" href="https://cdn.sstatic.net/Shared/stacks.css?v=1e9dfb1f6199"> <link rel="stylesheet" type="text/css" href="https://cdn.sstatic.net/Sites/matheducators/primary.css?v=fffd8c6b5303"> <link rel="alternate" type="application/atom+xml" title="Feed for question 'Inquiry about my note-taking skill'" href="/feeds/question/8456"> <script> StackExchange.ready(function () { StackExchange.using("postValidation", function () { StackExchange.postValidation.initOnBlurAndSubmit($('#post-form'), 2, 'answer'); }); StackExchange.question.init({showAnswerHelp:true,totalCommentCount:0,shownCommentCount:0,enableTables:true,questionId:8456}); styleCode(); StackExchange.realtime.subscribeToQuestion('548', '8456'); StackExchange.using("gps", function () { StackExchange.gps.trackOutboundClicks('#content', '.js-post-body'); }); }); </script> <link rel="stylesheet" type="text/css" href="https://cdn.sstatic.net/Shared/Channels/channels.css?v=5981bb1a5bd7"> <script type="text/x-mathjax-config"> MathJax.Hub.Config({ "HTML-CSS": { preferredFont: "TeX", availableFonts: ["STIX", "TeX"], linebreaks: { automatic: true }, EqnChunk: (MathJax.Hub.Browser.isMobile ? 10 : 50) }, tex2jax: { inlineMath: [["$", "$"], ["\\\\(","\\\\)"]], displayMath: [["$$", "$$"], ["\\[", "\\]"]], processEscapes: true, ignoreClass: "tex2jax_ignore|dno" }, TeX: { extensions: ["begingroup.js"], noUndefined: { attributes: { mathcolor: "red", mathbackground: "#FFEEEE", mathsize: "90%" } }, Macros: { href: "{}" } }, messageStyle: "none", styles: { ".MathJax_Display, .MathJax_Preview, .MathJax_Preview > *": { "background": "inherit" } }, SEEditor: "mathjaxEditing" }); </script> <script src="https://cdnjs.cloudflare.com/ajax/libs/mathjax/2.7.5/MathJax.js?config=TeX-AMS_HTML-full"></script> <script> StackExchange.ready(function () { StackExchange.realtime.init('wss://qa.sockets.stackexchange.com'); StackExchange.realtime.subscribeToReputationNotifications('548'); StackExchange.realtime.subscribeToTopBarNotifications('548'); }); </script> <script type="application/json" data-role="module-args" data-module-name="Shared/options.mod">{"options":{"locale":"en","serverTime":1735780817,"routeName":"Questions/Show","stackAuthUrl":"https://stackauth.com","networkMetaHostname":"meta.stackexchange.com","site":{"name":"Mathematics Educators Stack Exchange","description":"Q\u0026A for those involved in the field of teaching mathematics","isNoticesTabEnabled":true,"enableNewTagCreationWarning":false,"insertSpaceAfterNameTabCompletion":false,"id":548,"cookieDomain":".stackexchange.com","childUrl":"https://matheducators.meta.stackexchange.com","negativeVoteScoreFloor":null,"enableSocialMediaInSharePopup":true,"protocol":"https"},"user":{"fkey":"b39cb1a761f93912de068f8e6267ee5702b7d597a570755c5112354b099addd4","tid":"964cbf36-0399-487f-aa4f-9a8b062b7563","rep":0,"isAnonymous":true,"isAnonymousNetworkWide":true},"realtime":{"newest":true,"active":true,"tagged":true,"staleDisconnectIntervalInHours":0},"events":{"postType":{"question":1},"postEditionSection":{"title":1,"body":2,"tags":3}}}}</script> <script type="application/json" data-role="module-args" data-module-name="Shared/settings.mod">{"settings":{"image":{"maxImageUploadSizeInBytes":10485760,"maxImageUploadSizeInBytesAnimatedGif":2097152},"paths":{"jQueryUIJSPath":"https://ajax.googleapis.com/ajax/libs/jqueryui/1.12.0/jquery-ui.min.js","jQueryUICSSPath":"https://ajax.googleapis.com/ajax/libs/jqueryui/1.12.0/themes/smoothness/jquery-ui.css"},"comments":{},"userMessaging":{},"elections":{"opaVoteResultsBaseUrl":"https://www.opavote.com/results/"},"questions":{"enableQuestionTitleLengthLiveWarning":true,"enableSavesFeature":true,"questionTitleLengthStartLiveWarningChars":50,"maxTitleSize":150},"tags":{},"accounts":{"currentPasswordRequiredForChangingStackIdPassword":true},"site":{"enableUserHovercards":true,"enableImageHttps":true,"allowImageUploads":true,"forceHttpsImages":true,"stacksEditorPreviewEnabled":true},"flags":{"allowRetractingCommentFlags":true,"allowRetractingFlags":true},"snippets":{"renderDomain":"stacksnippets.net"},"intercom":{"appId":"inf0secd"},"auth":{"oauthInPopup":true},"subscriptions":{"defaultBasicMaxTrueUpSeats":250,"defaultMaxTrueUpSeats":1000,"defaultFreemiumMaxTrueUpSeats":50},"questionLinkTitleReplacement":{"maxNumberOfSitesProcessed":10,"maxReplacementsPerSite":20},"markdown":{"enableTables":true},"mentions":{"maxNumUsersInDropdown":50},"legal":{"useCustomConsent":false,"oneTrustTCFConfigId":"cb0f3c87-b769-4e66-bbaa-377f9194216d"},"search":{}}}</script> <script>StackExchange.init();</script> <script> StackExchange.using.setCacheBreakers({"Js/adops.en.js":"6da43f5e0a84","Js/ask.en.js":"","Js/begin-edit-event.en.js":"20edbaccceae","Js/copy-transpiled.en.js":"7959520085c5","Js/events.en.js":"","Js/explore-qlist.en.js":"ee2a4f8c3992","Js/full-anon.en.js":"756e9cf92803","Js/full.en.js":"db82f5b1e046","Js/highlightjs-loader.en.js":"dec53251ce5d","Js/inline-tag-editing.en.js":"8517756a2cb6","Js/keyboard-shortcuts.en.js":"c255a5a5979b","Js/markdown-it-loader.en.js":"5818ef89ff9d","Js/mentions-transpiled.en.js":"54b80f913964","Js/moderator.en.js":"038dfaeac3b5","Js/postCollections-transpiled.en.js":"fd1c4a681d04","Js/post-validation.en.js":"6c596a8d33b1","Js/question-editor.en.js":"","Js/review-v2-transpiled.en.js":"b80294337dec","Js/revisions.en.js":"9dd135bb585f","Js/stacks-editor.en.js":"8de4a63a68e8","Js/tageditor.en.js":"4d22c6090e5a","Js/tageditornew.en.js":"4554c63a5fa6","Js/tagsuggestions.en.js":"d9e40cbceb75","Js/unlimited-transpiled.en.js":"8713a979101d","Js/wmd.en.js":"8e5e21c8ea03","Js/mathjax-editing.v3.en.js":"4f38b81017dc","Js/mathjax-editing.en.js":"2581d730d948"}); StackExchange.using("gps", function() { StackExchange.gps.init(false); }); </script> <noscript id="noscript-css"><style>body,.s-topbar{margin-top:1.9em}</style></noscript> </head> <body class="question-page unified-theme"> <div id="signup-modal-container"></div> <script type="application/json" data-role="module-args" data-module-name="islands/signup-modal/index.mod">{"ContainerElementId":"signup-modal-container","FKey":"b39cb1a761f93912de068f8e6267ee5702b7d597a570755c5112354b099addd4","TriggerEvent":"signupModalShow","OauthInPopup":true,"ReturnUrl":"https://matheducators.stackexchange.com/questions/8456/inquiry-about-my-note-taking-skill/8475","ReturnUrlForPopup":"https://matheducators.stackexchange.com/users/after-signup/oauth-only","SiteName":"Mathematics Educators","SiteLogoPath":"https://cdn.sstatic.net/Sites/matheducators/Img/icon-48.png?v=814c297214f8","AuthProviders":["Google"],"ParentSiteUrl":"","IsInitiallyVisible":false}</script> <script defer src="https://cdn.sstatic.net/Js/webpack-chunks/svelte.en.js?v=150134e89426"></script><script defer src="https://cdn.sstatic.net/Js/webpack-chunks/stacks-svelte.en.js?v=72feec5d5528"></script><script defer src="https://cdn.sstatic.net/Js/webpack-chunks/1315.en.js?v=d971ebf7a8e2"></script><script defer src="https://cdn.sstatic.net/Js/webpack-chunks/4537.en.js?v=e6769247457b"></script><script defer src="https://cdn.sstatic.net/Js/islands/signup-modal.en.js?v=70d42243ade4"></script> <script defer> dispatchEvent(new CustomEvent("openSignupModal")); </script> <div id="one-tap-container"></div> <script type="application/json" data-role="module-args" data-module-name="islands/one-tap/index.mod">{"ContainerElementId":"one-tap-container","FKey":"b39cb1a761f93912de068f8e6267ee5702b7d597a570755c5112354b099addd4","GoogleClientId":"717762328687-iludtf96g1hinl76e4lc1b9a82g457nn.apps.googleusercontent.com","Autoselect":false,"ReturnUrl":"https%3a%2f%2fmatheducators.stackexchange.com%2fquestions%2f8456%2finquiry-about-my-note-taking-skill%2f8475"}</script><script defer src="https://cdn.sstatic.net/Js/webpack-chunks/svelte.en.js?v=150134e89426"></script><script defer src="https://cdn.sstatic.net/Js/islands/one-tap.en.js?v=661858832214"></script> <div id="notify-container"></div> <div id="custom-header"></div> <header class="s-topbar ps-fixed t0 l0 js-top-bar"> <a href="#content" class="s-topbar--skip-link">Skip to main content</a> <div class="s-topbar--container"> <a href="#" class="s-topbar--menu-btn js-left-sidebar-toggle" role="menuitem" aria-haspopup="true" aria-controls="left-sidebar" aria-expanded="false"><span></span></a> <div class="topbar-dialog leftnav-dialog js-leftnav-dialog dno"> <div class="left-sidebar js-unpinned-left-sidebar" data-can-be="left-sidebar" data-is-here-when="sm"></div> </div> <a href="#" class="s-topbar--logo network-logo js-gps-track js-network-logo" data-gps-track="stack_exchange_popup.show" role="menuitem" aria-haspopup="true" aria-controls="topbar-network-logo-dialog" aria-expanded="false"> <svg aria-hidden="true" class="native mtn1 svg-icon iconSEAlternativeSm" width="107" height="15" viewBox="0 0 107 15"><path fill="#FEFEFE" d="m48.41 11.93-1.96-3.2-1.04 1.16v2.04h-1.42V2.18h1.42v6.01L48.14 5h1.72l-2.44 2.7 2.74 4.22zm-7.06.08c-1.59 0-3.14-.96-3.14-3.56s1.55-3.54 3.14-3.54c.97 0 1.65.27 2.31.97l-.97.93c-.44-.48-.79-.66-1.34-.66q-.84.01-1.3.62c-.31.38-.42.87-.42 1.68s.1 1.32.41 1.7c.3.4.76.62 1.3.62.56 0 .9-.18 1.35-.66l.97.92c-.66.7-1.34.98-2.31.98m-5.66-3.15h-1.65c-.83 0-1.26.37-1.26 1s.4.99 1.3.99c.53 0 .93-.04 1.3-.4q.32-.29.31-1.03zm.03 3.07v-.63c-.51.5-1 .71-1.87.71s-1.46-.2-1.89-.63a2 2 0 0 1-.55-1.49c0-1.16.82-2 2.42-2h1.86v-.5c0-.87-.44-1.3-1.54-1.3-.77 0-1.15.18-1.54.68l-.92-.86c.66-.77 1.35-1 2.52-1q2.9 0 2.9 2.38v4.64zm-5.9 0c-1.32 0-1.93-.93-1.93-1.93V6.18h-.8V5.1h.8V3h1.41v2.1h1.36v1.07H29.3v3.75c0 .5.25.81.78.81h.58v1.2zm-6.33.08c-1.48 0-2.55-.34-3.49-1.28l1-.98c.72.72 1.51.94 2.52.94 1.3 0 2.04-.55 2.04-1.5q0-.65-.39-1.01c-.25-.23-.5-.33-1.08-.41l-1.16-.17a3.4 3.4 0 0 1-1.88-.78 2.4 2.4 0 0 1-.72-1.86c0-1.7 1.25-2.86 3.3-2.86 1.3 0 2.22.33 3.07 1.1l-.96.94a3 3 0 0 0-2.15-.75c-1.16 0-1.8.65-1.8 1.52q-.02.54.37.9c.25.22.65.38 1.11.45l1.13.17c.91.13 1.42.35 1.84.72.54.47.8 1.17.8 2 0 1.8-1.48 2.86-3.55 2.86"/><path fill="#2F96E8" d="M104.16 7.09c-.2-.42-.6-.74-1.2-.74s-.99.32-1.18.74c-.1.25-.15.44-.16.75h2.7a2 2 0 0 0-.16-.75m-2.54 1.96c0 .9.56 1.57 1.55 1.57.78 0 1.16-.21 1.61-.66l1.08 1.04a3.4 3.4 0 0 1-2.7 1.11c-1.68 0-3.29-.76-3.29-3.62 0-2.3 1.26-3.6 3.1-3.6 1.97 0 3.1 1.44 3.1 3.37v.79zm-5.48-2.57C95.1 6.48 95 7.37 95 8.3s.1 1.85 1.15 1.85 1.18-.91 1.18-1.85c0-.93-.13-1.82-1.18-1.82m-.17 8.22c-1.1 0-1.84-.21-2.58-.92l1.1-1.11c.4.38.8.54 1.4.54 1.06 0 1.43-.74 1.43-1.46v-.72c-.47.51-1 .7-1.7.7-.69 0-1.29-.23-1.68-.62-.67-.66-.73-1.57-.73-2.8 0-1.24.06-2.13.73-2.8.4-.39 1-.62 1.7-.62.75 0 1.24.2 1.73.75v-.67h1.72v6.8c0 1.7-1.21 2.93-3.12 2.93m-5.76-2.67V7.76c0-.96-.61-1.28-1.17-1.28s-1.18.32-1.18 1.28v4.27h-1.78V4.97h1.73v.65a2.4 2.4 0 0 1 1.78-.73q1.07.02 1.67.62c.58.57.73 1.24.73 2v4.52zm-7.1-2.98h-1.4c-.64 0-1 .3-1 .8 0 .49.33.81 1.02.81.5 0 .8-.04 1.12-.34q.28-.25.26-.89zm.04 2.98v-.6c-.48.47-.93.67-1.74.67q-1.2 0-1.82-.62c-.38-.4-.58-.97-.58-1.59 0-1.12.77-2.05 2.42-2.05h1.68V7.5c0-.77-.38-1.11-1.32-1.11-.68 0-1 .16-1.37.58l-1.13-1.1c.7-.75 1.38-.97 2.57-.97q3 0 3.02 2.5v4.64zm-6.93 0v-4.3c0-.94-.6-1.25-1.15-1.25-.56 0-1.15.32-1.15 1.24v4.31h-1.77V2.38h1.77v3.24a2.4 2.4 0 0 1 1.7-.73c1.56 0 2.38 1.08 2.38 2.57v4.57zm-6.96.08c-1.42 0-3.18-.76-3.18-3.62 0-2.85 1.76-3.6 3.18-3.6.98 0 1.72.3 2.34.95l-1.2 1.2c-.36-.4-.68-.56-1.14-.56q-.62-.01-1.01.46c-.27.33-.4.8-.4 1.55s.13 1.24.4 1.58q.39.46 1 .46c.47 0 .79-.16 1.15-.56l1.2 1.18c-.62.65-1.36.96-2.34.96m-5.53-.08-1.3-2.11-1.3 2.11H59l2.45-3.6-2.35-3.46h2.12L62.42 7l1.21-2.02h2.13L63.4 8.43l2.46 3.6zm-11.75 0V2.06h6.6V3.8h-4.65v2.33h3.96v1.74h-3.96v2.42h4.65v1.74z"/><path fill="#8FD8F7" d="M0 3c0-1.1.9-2 2-2h8a2 2 0 0 1 2 2z"/><path fill="#155397" d="M12 10H0c0 1.1.9 2 2 2h5v3l3-3a2 2 0 0 0 2-2"/><path fill="#46A2D9" d="M0 4h12v2H0z"/><path fill="#2D6DB5" d="M0 7h12v2H0z"/></svg> </a> <div class="topbar-dialog network-logo-dialog js-network-logo-dialog dno" id="topbar-network-logo-dialog" role="dialog" aria-labelledby="topbar-network-logo-dialog-title" aria-describedby="topbar-network-logo-dialog-body"> <div class="dialog-content"> <h4 class="bold" id="topbar-network-logo-dialog-title">Stack Exchange Network</h4> <p id="topbar-network-logo-dialog-body"> Stack Exchange network consists of 183 Q&A communities including <a href="https://stackoverflow.com">Stack Overflow</a>, the largest, most trusted online community for developers to learn, share their knowledge, and build their careers. </p> <a class="s-btn s-btn__filled" href="https://stackexchange.com" data-gps-track="stack_exchange_popup.click">Visit Stack Exchange</a> <button class="s-btn s-btn__muted p0 ps-absolute t16 r16 js-close-button" aria-label="Close"><svg aria-hidden="true" class="svg-icon iconClear" width="18" height="18" viewBox="0 0 18 18"><path d="M15 4.41 13.59 3 9 7.59 4.41 3 3 4.41 7.59 9 3 13.59 4.41 15 9 10.41 13.59 15 15 13.59 10.41 9z"/></svg></button> </div> </div> <form id="search" role="search" action=/search class="s-topbar--searchbar js-searchbar " autocomplete="off"> <div class="s-topbar--searchbar--input-group"> <input name="q" type="text" role="combobox" placeholder="Search on Mathematics Educators…" value="" autocomplete="off" maxlength="240" class="s-input s-input__search js-search-field wmn1 " aria-label="Search" aria-controls="top-search" data-controller="s-popover" data-action="focus->s-popover#show" data-s-popover-placement="bottom-start" /> <svg aria-hidden="true" class="s-input-icon s-input-icon__search svg-icon iconSearch" width="18" height="18" viewBox="0 0 18 18"><path d="m18 16.5-5.14-5.18h-.35a7 7 0 1 0-1.19 1.19v.35L16.5 18zM12 7A5 5 0 1 1 2 7a5 5 0 0 1 10 0"/></svg> <div class="s-popover p0 wmx100 wmn4 sm:wmn-initial js-top-search-popover" id="top-search" role="menu"> <div class="s-popover--arrow"></div> <div class="s-popover--content"> <div class="js-spinner p24 d-flex ai-center jc-center d-none"> <div class="s-spinner s-spinner__sm fc-orange-400"> <div class="v-visible-sr">Loading…</div> </div> </div> <span class="v-visible-sr js-screen-reader-info"></span> <div class="js-ac-results overflow-y-auto hmx3 d-none"></div> <div class="js-search-hints" aria-describedby="Tips for searching"></div> </div> </div> </div> </form> <nav class="h100 ml-auto overflow-x-auto pr12" aria-label="Topbar"> <ol class="s-topbar--content" role="menubar"> <li role="none"> <a href="/help" class="s-topbar--item js-help-button" role="menuitem" title="Help Center and other resources" aria-haspopup="true" aria-controls="topbar-help-dialog" data-ga="["top navigation","help menu click",null,null,null]"><svg aria-hidden="true" class="svg-icon iconHelp" width="18" height="18" viewBox="0 0 18 18"><path d="M9 1C4.64 1 1 4.64 1 9s3.64 8 8 8 8-3.64 8-8-3.64-8-8-8m.81 12.13c-.02.71-.55 1.15-1.24 1.13-.66-.02-1.17-.49-1.15-1.2.02-.72.56-1.18 1.22-1.16.7.03 1.2.51 1.17 1.23M11.77 8c-.59.66-1.78 1.09-2.05 1.97a4 4 0 0 0-.09.75c0 .05-.03.16-.18.16H7.88c-.16 0-.18-.1-.18-.15.06-1.35.66-2.2 1.83-2.88.39-.29.7-.75.7-1.24.01-1.24-1.64-1.82-2.35-.72-.21.33-.18.73-.18 1.1H5.75c0-1.97 1.03-3.26 3.03-3.26 1.75 0 3.47.87 3.47 2.83 0 .57-.2 1.05-.48 1.44"/></svg></a> </li> <div class="topbar-dialog help-dialog js-help-dialog dno" id="topbar-help-dialog" role="menu"> <div class="modal-content"> <ul> <li> <a href="/tour" class="js-gps-track s-block-link" data-gps-track="help_popup.click({ item_type:1 })" data-ga="["top navigation","tour submenu click",null,null,null]"> Tour <span class="item-summary"> Start here for a quick overview of the site </span> </a> </li> <li> <a href="/help" class="js-gps-track s-block-link" data-gps-track="help_popup.click({ item_type:4 })" data-ga="["top navigation","help center",null,null,null]"> Help Center <span class="item-summary"> Detailed answers to any questions you might have </span> </a> </li> <li> <a href="https://matheducators.meta.stackexchange.com" class="js-gps-track s-block-link" data-gps-track="help_popup.click({ item_type:2 })" data-ga="["top navigation","meta submenu click",null,null,null]"> Meta <span class="item-summary"> Discuss the workings and policies of this site </span> </a> </li> <li> <a href="https://stackoverflow.co/" class="js-gps-track s-block-link" data-gps-track="help_popup.click({ item_type:6 })" data-ga="["top navigation","about us submenu click",null,null,null]"> About Us <span class="item-summary"> Learn more about Stack Overflow the company, and our products </span> </a> </li> </ul> </div> </div> <li role="none"> <a href="https://stackexchange.com" class="s-topbar--item js-site-switcher-button js-gps-track" data-gps-track="site_switcher.show" aria-label="Site switcher" role="menuitem" title="A list of all 183 Stack Exchange sites" aria-haspopup="true" aria-expanded="false" data-ga="["top navigation","stack exchange click",null,null,null]"> <svg aria-hidden="true" class="svg-icon iconStackExchange" width="18" height="18" viewBox="0 0 18 18"><path d="M15 1H3a2 2 0 0 0-2 2v2h16V3a2 2 0 0 0-2-2M1 13c0 1.1.9 2 2 2h8v3l3-3h1a2 2 0 0 0 2-2v-2H1zm16-7H1v4h16z"/></svg> </a> </li> <li class="js-topbar-dialog-corral" role="presentation"> <div class="topbar-dialog siteSwitcher-dialog dno" role="menu"> <div class="header fw-wrap"> <h3 class="flex--item"> <a href="https://matheducators.stackexchange.com">current community</a> </h3> <div class="flex--item fl1"> <div class="ai-center d-flex jc-end"> <button class="js-close-button s-btn s-btn__muted p0 ml8 d-none sm:d-block" type="button" aria-label="Close" > <svg aria-hidden="true" class="svg-icon iconClear" width="18" height="18" viewBox="0 0 18 18"><path d="M15 4.41 13.59 3 9 7.59 4.41 3 3 4.41 7.59 9 3 13.59 4.41 15 9 10.41 13.59 15 15 13.59 10.41 9z"/></svg> </button> </div> </div> </div> <div class="modal-content bg-blue-200 current-site-container"> <ul class="current-site"> <li class="d-flex"> <div class="fl1"> <a href="https://matheducators.stackexchange.com" class="current-site-link d-flex gx8 site-link js-gps-track" data-id="548" data-gps-track="site_switcher.click({ item_type:3 })"> <div class="favicon favicon-matheducators site-icon flex--item" title="Mathematics Educators"></div> <span class="flex--item fl1"> Mathematics Educators </span> </a> </div> <div class="related-links"> <a href="https://matheducators.stackexchange.com/help" class="js-gps-track" data-gps-track="site_switcher.click({ item_type:14 })">help</a> <a href="https://chat.stackexchange.com?tab=site&host=matheducators.stackexchange.com" class="js-gps-track" data-gps-track="site_switcher.click({ item_type:6 })">chat</a> </div> </li> <li class="related-site d-flex"> <div class="L-shaped-icon-container"> <span class="L-shaped-icon"></span> </div> <a href="https://matheducators.meta.stackexchange.com" class="s-block-link px16 d-flex gx8 site-link js-gps-track" data-id="549" data-gps-track="site.switch({ target_site:549, item_type:3 }),site_switcher.click({ item_type:4 })"> <div class="favicon favicon-matheducatorsmeta site-icon flex--item" title="Mathematics Educators Meta"></div> <span class="flex--item fl1"> Mathematics Educators Meta </span> </a> </li> </ul> </div> <div class="header" id="your-communities-header"> <h3> your communities </h3> </div> <div class="modal-content" id="your-communities-section"> <div class="call-to-login"> <a href="https://matheducators.stackexchange.com/users/signup?ssrc=site_switcher&returnurl=https%3a%2f%2fmatheducators.stackexchange.com%2fquestions%2f8456%2finquiry-about-my-note-taking-skill%2f8475" class="login-link js-gps-track" data-gps-track="site_switcher.click({ item_type:10 })">Sign up</a> or <a href="https://matheducators.stackexchange.com/users/login?ssrc=site_switcher&returnurl=https%3a%2f%2fmatheducators.stackexchange.com%2fquestions%2f8456%2finquiry-about-my-note-taking-skill%2f8475" class="login-link js-gps-track" data-gps-track="site_switcher.click({ item_type:11 })">log in</a> to customize your list. </div> </div> <div class="header"> <h3><a href="https://stackexchange.com/sites">more stack exchange communities</a> </h3> <a href="https://stackoverflow.blog" class="float-right">company blog</a> </div> <div class="modal-content"> <div class="child-content"></div> </div> </div> </li> <li role="none"><button class="s-topbar--item s-btn s-btn__icon s-btn__muted d-none sm:d-inline-flex js-searchbar-trigger" role="menuitem" aria-label="Search" aria-haspopup="true" aria-controls="search" title="Click to show search"><svg aria-hidden="true" class="svg-icon iconSearch" width="18" height="18" viewBox="0 0 18 18"><path d="m18 16.5-5.14-5.18h-.35a7 7 0 1 0-1.19 1.19v.35L16.5 18zM12 7A5 5 0 1 1 2 7a5 5 0 0 1 10 0"/></svg></button></li> <li role="none"> <a href="https://matheducators.stackexchange.com/users/login?ssrc=head&returnurl=https%3a%2f%2fmatheducators.stackexchange.com%2fquestions%2f8456%2finquiry-about-my-note-taking-skill%2f8475" class="s-topbar--item s-topbar--item__unset s-btn s-btn__outlined ws-nowrap js-gps-track" role="menuitem" rel="nofollow" data-gps-track="login.click" data-ga="["top navigation","login button click",null,null,null]">Log in</a> </li> <li role="none"><a href="https://matheducators.stackexchange.com/users/signup?ssrc=head&returnurl=https%3a%2f%2fmatheducators.stackexchange.com%2fquestions%2f8456%2finquiry-about-my-note-taking-skill%2f8475" class="s-topbar--item s-topbar--item__unset ml4 s-btn s-btn__filled ws-nowrap js-signup-button js-gps-track" role="menuitem" rel="nofollow" data-gps-track="signup.topbar.click" data-ga="["sign up","Sign Up Navigation","Header",null,null]">Sign up</a></li> </ol> </nav> </div> </header> <script> StackExchange.ready(function () { StackExchange.topbar.init(); }); StackExchange.scrollPadding.setPaddingTop(50, 10); </script> <header class="site-header"> <div class="site-header--container"> <a class="site-header--link fs-headline1 fw-bold" href="https://matheducators.stackexchange.com"> Mathematics Educators </a> </div> </header> <div class="container"> <div id="left-sidebar" data-is-here-when="md lg" class="left-sidebar js-pinned-left-sidebar ps-relative"> <div class="left-sidebar--sticky-container js-sticky-leftnav"> <nav aria-label="Primary"> <ol class="nav-links"> <li> <ol class="nav-links"> <li class="ps-relative" aria-current="false"> <a href="/" class="s-block-link pl8 js-gps-track nav-links--link -link__with-icon" data-gps-track="top_nav.click({is_current: false, location:2, destination:8, has_activity_notification:False});home_nav.click({location:2})" aria-controls="" data-controller=" " data-s-popover-placement="right" aria-current="false" data-s-popover-auto-show="true" data-s-popover-hide-on-outside-click="never" > <div class="d-flex ai-center"> <svg aria-hidden="true" class="svg-icon iconHome" width="18" height="18" viewBox="0 0 18 18"><path d="M15 10v5a2 2 0 0 1-2 2H5a2 2 0 0 1-2-2v-5H0l9-9 9 9zm-8 1v6h4v-6z"/></svg> <span class="-link--channel-name pl6">Home</span> </div> </a> </li> <li class="ps-relative youarehere" aria-current="true"> <a id="nav-questions" href="/questions" class="s-block-link pl8 js-gps-track nav-links--link -link__with-icon" data-gps-track="top_nav.click({is_current: true, location:2, destination:1, has_activity_notification:False})" aria-controls="" data-controller=" " data-s-popover-placement="right" aria-current="false" data-s-popover-auto-show="true" data-s-popover-hide-on-outside-click="never" > <div class="d-flex ai-center"> <svg aria-hidden="true" class="svg-icon iconQuestion" width="18" height="18" viewBox="0 0 18 18"><path d="m4 15-3 3V4c0-1.1.9-2 2-2h12c1.09 0 2 .91 2 2v9c0 1.09-.91 2-2 2zm7.75-3.97c.72-.83.98-1.86.98-2.94 0-1.65-.7-3.22-2.3-3.83a4.4 4.4 0 0 0-3.02 0 3.8 3.8 0 0 0-2.32 3.83q0 1.93 1.03 3a3.8 3.8 0 0 0 2.85 1.07q.94 0 1.71-.34.97.66 1.06.7.34.2.7.3l.59-1.13a5 5 0 0 1-1.28-.66m-1.27-.9a5 5 0 0 0-1.5-.8l-.45.9q.5.18.98.5-.3.1-.65.11-.92 0-1.52-.68c-.86-1-.86-3.12 0-4.11.8-.9 2.35-.9 3.15 0 .9 1.01.86 3.03-.01 4.08"/></svg> <span class="-link--channel-name pl6">Questions</span> </div> </a> </li> <li class="ps-relative" aria-current="false"> <a href="/tags" class="s-block-link pl8 js-gps-track nav-links--link -link__with-icon" data-gps-track="top_nav.click({is_current: false, location:2, destination:2, has_activity_notification:False})" aria-controls="" data-controller=" " data-s-popover-placement="right" aria-current="false" data-s-popover-auto-show="true" data-s-popover-hide-on-outside-click="never" > <div class="d-flex ai-center"> <svg aria-hidden="true" class="svg-icon iconTags" width="18" height="18" viewBox="0 0 18 18"><path d="M9.24 1a3 3 0 0 0-2.12.88l-5.7 5.7a2 2 0 0 0-.38 2.31 3 3 0 0 1 .67-1.01l6-6A3 3 0 0 1 9.83 2H14a3 3 0 0 1 .79.1A2 2 0 0 0 13 1z" opacity=".4"/><path d="M9.83 3a2 2 0 0 0-1.42.59l-6 6a2 2 0 0 0 0 2.82L6.6 16.6a2 2 0 0 0 2.82 0l6-6A2 2 0 0 0 16 9.17V5a2 2 0 0 0-2-2zM12 9a2 2 0 1 1 0-4 2 2 0 0 1 0 4"/></svg> <span class="-link--channel-name pl6">Tags</span> </div> </a> </li> <li class="pb24"></li> <li class="ps-relative" aria-current="false"> <a id="nav-users" href="/users" class="s-block-link pl8 js-gps-track nav-links--link -link__with-icon" data-gps-track="top_nav.click({is_current: false, location:2, destination:3, has_activity_notification:False})" aria-controls="" data-controller=" " data-s-popover-placement="right" aria-current="false" data-s-popover-auto-show="true" data-s-popover-hide-on-outside-click="never" > <div class="d-flex ai-center"> <svg aria-hidden="true" class="svg-icon iconPeople" width="18" height="18" viewBox="0 0 18 18"><path d="M17 14c0 .44-.45 1-1 1H9a1 1 0 0 1-1-1H2c-.54 0-1-.56-1-1 0-2.63 3-4 3-4s.23-.4 0-1c-.84-.62-1.06-.59-1-3s1.37-3 2.5-3 2.44.58 2.5 3-.16 2.38-1 3c-.23.59 0 1 0 1s1.55.71 2.42 2.09c.78-.72 1.58-1.1 1.58-1.1s.23-.4 0-1c-.84-.61-1.06-.58-1-3s1.37-3 2.5-3 2.44.59 2.5 3c.05 2.42-.16 2.39-1 3-.23.6 0 1 0 1s3 1.38 3 4"/></svg> <span class="-link--channel-name pl6">Users</span> </div> </a> </li> <li class="ps-relative" aria-current="false"> <a id="nav-unanswered" href="/unanswered" class="s-block-link pl8 js-gps-track nav-links--link -link__with-icon" data-gps-track="top_nav.click({is_current: false, location:2, destination:5, has_activity_notification:False})" aria-controls="" data-controller=" " data-s-popover-placement="right" aria-current="false" data-s-popover-auto-show="true" data-s-popover-hide-on-outside-click="never" > <div class="d-flex ai-center"> <svg aria-hidden="true" class="svg-icon iconAnswer" width="18" height="18" viewBox="0 0 18 18"><path d="M14 15H3c-1.09 0-2-.91-2-2V4c0-1.1.9-2 2-2h12c1.09 0 2 .91 2 2v14zm-1.02-3L9.82 4H8.14l-3.06 8h1.68l.65-1.79h3.15l.69 1.79zm-2.93-3.12H7.9l1.06-2.92z"/></svg> <span class="-link--channel-name pl6">Unanswered</span> </div> </a> </li> </ol> </li> <li class="js-freemium-cta ps-relative mt32 mb8"> <div class="fs-fine tt-uppercase fc-black-600 fw-bold ml8 mt16 mb8">Teams</div> <div class="px12 pt12 pb4 mb12 fc-medium overflow-hidden"> <img class="wmx100 mx-auto mb12 h-auto d-block" width="151" height="24" src="https://cdn.sstatic.net/Img/teams/teams-promo.svg?v=e507948b81bf" alt=""> <p class="fs-fine"> Ask questions, find answers and collaborate at work with Stack Overflow for Teams. </p> <a href="https://stackoverflowteams.com/teams/create/free/?utm_medium=referral&utm_source=matheducators-community&utm_campaign=side-bar&utm_content=explore-teams" class="w100 s-btn s-btn__filled s-btn__xs bg-orange-400 h:bg-orange-500 js-gps-track pt8 pr7 pb6 pl7" data-gps-track="teams.create.left-sidenav.click({ Action: 6 })" data-ga="["teams left navigation - anonymous","left nav free cta","stackoverflow.com/teams/create/free",null,null]">Try Teams for free</a> <a href="https://stackoverflow.co/teams/?utm_medium=referral&utm_source=matheducators-community&utm_campaign=side-bar&utm_content=explore-teams" class="w100 s-btn s-btn__muted s-btn__xs mt1 js-gps-track" data-gps-track="teams.create.left-sidenav.click({ Action: 5 })" data-ga="["teams left navigation - anonymous","left nav free cta","stackoverflow.com/teams",null,null]">Explore Teams</a> </div> </li> <li class="d-flex ai-center jc-space-between ml8 mt32 mb8 js-create-team-cta d-none"> <a href="javascript:void(0)" class="s-link d-flex fl-grow1 fc-black-400 h:fc-black-600 fs-fine js-gps-track" role="button" aria-controls="popover-teams-create-cta" data-controller="s-popover" data-action="s-popover#toggle" data-s-popover-placement="bottom-start" data-s-popover-toggle-class="is-selected" data-gps-track="teams.create.left-sidenav.click({ Action: ShowInfo })" data-ga="["teams left navigation - anonymous","left nav show teams info",null,null,null]" > <div class="flex--item fl-grow1 fc-black-600 fw-bold tt-uppercase">Teams</div> <div class="flex--item px12"> <svg aria-hidden="true" class="svg-icon iconPlusSm" width="14" height="14" viewBox="0 0 14 14"><path d="M8 2H6v4H2v2h4v4h2V8h4V6H8z"/></svg> </div> </a> </li> <li class="ps-relative js-create-team-cta d-none"> <p class="fs-fine pr8 pl8 pb4 fc-black-400"> Ask questions, find answers and collaborate at work with Stack Overflow for Teams. <a href="https://stackoverflow.co/teams/?utm_medium=referral&utm_source=matheducators-community&utm_campaign=side-bar&utm_content=explore-teams-compact" class="s-link s-link__grayscale s-link__underlined fw-bold">Explore Teams</a> </p> </li> </ol> </nav> </div> <div class="s-popover ws2" id="popover-teams-create-cta" role="menu" aria-hidden="true"> <div class="s-popover--arrow"></div> <div class="ps-relative overflow-hidden"> <p class="mb2"><strong>Teams</strong></p> <p class="mb12 fs-caption fc-black-400">Q&A for work</p> <p class="mb12 fs-caption fc-black-500">Connect and share knowledge within a single location that is structured and easy to search.</p> <a href="https://stackoverflow.co/teams/" class="js-gps-track s-btn s-btn__filled s-btn__xs" data-gps-track="teams.create.left-sidenav.click({ Action: CtaClick })" data-ga="["teams left navigation - anonymous","left nav cta","stackoverflow.com/teams",null,null]"> Learn more about Teams </a> </div> <div class="ps-absolute t8 r8"> <svg aria-hidden="true" class="fc-orange-400 svg-spot spotPeople" width="48" height="48" viewBox="0 0 48 48"><path d="M13.5 28a4.5 4.5 0 1 0 0-9 4.5 4.5 0 0 0 0 9M7 30a1 1 0 0 1 1-1h11a1 1 0 0 1 1 1v5h11v-5a1 1 0 0 1 1-1h12a1 1 0 0 1 1 1v10a2 2 0 0 1-2 2H33v5a1 1 0 0 1-1 1H20a1 1 0 0 1-1-1v-5H8a1 1 0 0 1-1-1zm25-6.5a4.5 4.5 0 1 0 9 0 4.5 4.5 0 0 0-9 0M24.5 34a4.5 4.5 0 1 0 0-9 4.5 4.5 0 0 0 0 9" opacity=".2"/><path d="M16.4 26.08A6 6 0 1 0 7.53 26C5.64 26.06 4 27.52 4 29.45V40a1 1 0 0 0 1 1h9a1 1 0 1 0 0-2h-4v-7a1 1 0 1 0-2 0v7H6v-9.55c0-.73.67-1.45 1.64-1.45H16a1 1 0 0 0 .4-1.92M12 18a4 4 0 1 1 0 8 4 4 0 0 1 0-8m16.47 14a6 6 0 1 0-8.94 0A3.6 3.6 0 0 0 16 35.5V46a1 1 0 0 0 1 1h14a1 1 0 0 0 1-1V35.5c0-1.94-1.64-3.42-3.53-3.5M20 28a4 4 0 1 1 8 0 4 4 0 0 1-8 0m-.3 6h8.6c1 0 1.7.75 1.7 1.5V45h-2v-7a1 1 0 1 0-2 0v7h-4v-7a1 1 0 1 0-2 0v7h-2v-9.5c0-.75.7-1.5 1.7-1.5M42 22c0 1.54-.58 2.94-1.53 4A3.5 3.5 0 0 1 44 29.45V40a1 1 0 0 1-1 1h-9a1 1 0 1 1 0-2h4v-7a1 1 0 1 1 2 0v7h2v-9.55A1.5 1.5 0 0 0 40.48 28H32a1 1 0 0 1-.4-1.92A6 6 0 1 1 42 22m-2 0a4 4 0 1 0-8 0 4 4 0 0 0 8 0"/><g opacity=".35"><path d="M17 10a1 1 0 011-1h12a1 1 0 110 2H18a1 1 0 01-1-1m1-5a1 1 0 100 2h12a1 1 0 100-2zM14 1a1 1 0 00-1 1v12a1 1 0 001 1h5.09l4.2 4.2a1 1 0 001.46-.04l3.7-4.16H34a1 1 0 001-1V2a1 1 0 00-1-1zm1 12V3h18v10h-5a1 1 0 00-.75.34l-3.3 3.7-3.74-3.75a1 1 0 00-.71-.29z"/></g></svg> </div> </div> </div> <div id="content" class=""> <div itemprop="mainEntity" itemscope itemtype="https://schema.org/Question"> <link itemprop="image" href="https://cdn.sstatic.net/Sites/matheducators/Img/apple-touch-icon.png?v=41c548fc9c75"> <div class="inner-content clearfix"> <div id="question-header" class="d-flex sm:fd-column"> <h1 itemprop="name" class="fs-headline1 ow-break-word mb8 flex--item fl1"><a href="/questions/8456/inquiry-about-my-note-taking-skill" class="question-hyperlink">Inquiry about my note-taking skill</a></h1> <div class="ml12 aside-cta flex--item sm:ml0 sm:mb12 sm:order-first d-flex jc-end"> <div class="ml12 aside-cta flex--item print:d-none"> <a href="/questions/ask" class="ws-nowrap s-btn s-btn__filled"> Ask Question </a> </div> </div> </div> <div class="d-flex fw-wrap pb8 mb16 bb bc-black-200"> <div class="flex--item ws-nowrap mr16 mb8" title="2015-07-31 19:05:46Z"> <span class="fc-black-400 mr2">Asked</span> <time itemprop="dateCreated" datetime="2015-07-31T19:05:46">9 years, 5 months ago</time> </div> <div class="flex--item ws-nowrap mr16 mb8"> <span class="fc-black-400 mr2">Modified</span> <a href="?lastactivity" class="s-link s-link__inherit" title="2016-01-25 19:11:36Z">8 years, 11 months ago</a> </div> <div class="flex--item ws-nowrap mb8" title="Viewed 1,449 times"> <span class="fc-black-400 mr2">Viewed</span> 1k times </div> </div> <div id="mainbar" role="main" aria-label="question and answers"> <div class="question js-question" data-questionid="8456" data-position-on-page="0" data-score="18" id="question"> <style> </style> <div class="js-zone-container zone-container-main"> <div id="dfp-tlb" class="everyonelovesstackoverflow everyoneloves__top-leaderboard everyoneloves__leaderboard"></div> <div class="js-report-ad-button-container " style="width: 728px"></div> </div> <div class="post-layout "> <div class="votecell post-layout--left"> <div class="js-voting-container d-flex jc-center fd-column ai-center gs4 fc-black-300" data-post-id="8456" data-referrer="None"> <button class="js-vote-up-btn flex--item s-btn s-btn__muted s-btn__outlined bar-pill bc-black-225 f:bc-theme-secondary-400 f:bg-theme-secondary-400 f:fc-black-050 h:bg-theme-primary-200" data-controller="s-tooltip" data-s-tooltip-placement="right" title="This question shows research effort; it is useful and clear" aria-pressed="false" aria-label="Up vote" data-selected-classes="fc-theme-primary-100 bc-theme-primary-500 bg-theme-primary-500" data-unselected-classes="bc-black-225 f:bc-theme-secondary-400 f:bg-theme-secondary-400 f:fc-black-050 h:bg-theme-primary-200"> <svg aria-hidden="true" class="svg-icon iconArrowUp" width="18" height="18" viewBox="0 0 18 18"><path d="M1 12h16L9 4z"/></svg> </button> <input type="hidden" id="voteUpHash" value="66:3:31e,16:4cfe873d5a64b3a3,10:1735780817,16:9b88ff2a04b1f122,4:8456,1fa94ae4e05da89e4b496689d63851d0a2f51acb9e83fecc361cbde7490996c9" /> <div class="js-vote-count flex--item d-flex fd-column ai-center fc-theme-body-font fw-bold fs-subheading py4" itemprop="upvoteCount" data-value="18"> 18 </div> <button class="js-vote-down-btn js-vote-down-question flex--item mb8 s-btn s-btn__muted s-btn__outlined bar-pill bc-black-225 f:bc-theme-secondary-400 f:bg-theme-secondary-400 f:fc-black-050 h:bg-theme-primary-200" title="This question does not show any research effort; it is unclear or not useful" aria-pressed="false" aria-label="Down vote" data-selected-classes="fc-theme-primary-100 bc-theme-primary-500 bg-theme-primary-500" data-unselected-classes="bc-black-225 f:bc-theme-secondary-400 f:bg-theme-secondary-400 f:fc-black-050 h:bg-theme-primary-200"> <svg aria-hidden="true" class="svg-icon iconArrowDown" width="18" height="18" viewBox="0 0 18 18"><path d="M1 6h16l-8 8z"/></svg> </button> <input type="hidden" id="voteDownHash" value="66:3:31e,16:d45183bab8f3c074,10:1735780817,16:9335b5b7210c2395,4:8456,a8adfa8e337192f3b30a53a53031b943dd3c24e483f88935c931cd7aed4e7e34" /> <button class="js-saves-btn s-btn s-btn__unset c-pointer py4" type="button" id="saves-btn-8456" data-controller="s-tooltip" data-s-tooltip-placement="right" data-s-popover-placement="" title="Save this question." data-is-saved="false" aria-label="Save" data-post-id="8456" data-post-type-id="1" data-user-privilege-for-post-click="0" aria-controls="" data-s-popover-auto-show="false" > <svg aria-hidden="true" class="fc-theme-primary-400 js-saves-btn-selected d-none svg-icon iconBookmark" width="18" height="18" viewBox="0 0 18 18"><path d="M3 17V3c0-1.1.9-2 2-2h8a2 2 0 0 1 2 2v14l-6-4z"/></svg> <svg aria-hidden="true" class="js-saves-btn-unselected svg-icon iconBookmarkAlt" width="18" height="18" viewBox="0 0 18 18"><path d="m9 10.6 4 2.66V3H5v10.26zM3 17V3c0-1.1.9-2 2-2h8a2 2 0 0 1 2 2v14l-6-4z"/></svg> </button> <a class="js-post-issue flex--item s-btn s-btn__unset c-pointer py6 mx-auto" href="/posts/8456/timeline" data-shortcut="T" data-ks-title="timeline" data-controller="s-tooltip" data-s-tooltip-placement="right" title="Show activity on this post." aria-label="Timeline"><svg aria-hidden="true" class="mln2 mr0 svg-icon iconHistory" width="19" height="18" viewBox="0 0 19 18"><path d="M3 9a8 8 0 1 1 3.73 6.77L8.2 14.3A6 6 0 1 0 5 9l3.01-.01-4 4-4-4zm7-4h1.01L11 9.36l3.22 2.1-.6.93L10 10z"/></svg></a> </div> </div> <div class="postcell post-layout--right"> <span class="d-none">$\begingroup$</span> <div class="s-prose js-post-body" itemprop="text"> <p>I am a rising college junior in US with a major in mathematics. I recently noticed a problem in my note-taking skill in the mathematics, both from the textbooks and lectures. When I was a microbiology major, I wrote extensive amount of notes from my books and lectures since I had to memorize all of them. I followed this trend and took extensive notes from my math books and professors' lectures. The problem with my note-taking skill is that I basically copy down almost everything from my books. For example, I am currently studying Apostol's Mathematical Analysis; what I am doing is that I copy down the theorems, proofs (including my footnotes), authors's remarks & motivation, and examples. I tried to copy them on my own words, but the result is that I basically copy down verbatim with little change in the grammars. I started to realized that I should not take such exhaustive notes since the textbooks contain exact materials from my notebooks, but I cannot overcome the feeling that I need to make my own version of books, by taking notes from them, and the fear of memory loss. Also I tried to take as much notes as possible from the professors's lectures. What I realized is that I copy down the exact words from my professors, and I often lost the concentration to focus on lectures and absorb the materials. I also lost my concentration whenever I take the notes from my books since I need to divert my attention from books to notebooks. </p> <p>I am currently thinking about various alternative strategies, which are followings:</p> <p>1) Taking notes within the textbook using the blank space of books and the Post-It: Take notes about interesting remarks, confusion about the exposition and examples, and my own interesting ideas or approach to the proofs and examples.</p> <p>2) Taking notes on a separate notebook but only copying the information from 1). </p> <p>3) Do not take any notes and try to absorb the materials from the books: I noticed that I usually learn the best by reading. Use the lecture to supplement the textbook reading and to gain different perspective.</p> <p>Could you help me out by commenting about my strategies and/or share your note-taking skills? I am quite embarrassed about my note-taking problem.</p> </div> <div class="mt24 mb12"> <div class="post-taglist d-flex gs4 gsy fd-column"> <div class="d-flex ps-relative fw-wrap"> <ul class='ml0 list-ls-none js-post-tag-list-wrapper d-inline'><li class='d-inline mr4 js-post-tag-list-item'><a href="/questions/tagged/undergraduate-education" class="s-tag post-tag" title="show questions tagged 'undergraduate-education'" aria-label="show questions tagged 'undergraduate-education'" rel="tag" aria-labelledby="tag-undergraduate-education-tooltip-container" data-tag-menu-origin="Unknown">undergraduate-education</a></li><li class='d-inline mr4 js-post-tag-list-item'><a href="/questions/tagged/self-learning" class="s-tag post-tag" title="show questions tagged 'self-learning'" aria-label="show questions tagged 'self-learning'" rel="tag" aria-labelledby="tag-self-learning-tooltip-container" data-tag-menu-origin="Unknown">self-learning</a></li><li class='d-inline mr4 js-post-tag-list-item'><a href="/questions/tagged/lecture-notes" class="s-tag post-tag" title="show questions tagged 'lecture-notes'" aria-label="show questions tagged 'lecture-notes'" rel="tag" aria-labelledby="tag-lecture-notes-tooltip-container" data-tag-menu-origin="Unknown">lecture-notes</a></li></ul> </div> </div> </div> <div class="mb0 "> <div class="mt16 d-flex gs8 gsy fw-wrap jc-end ai-start pt4 mb16"> <div class="flex--item mr16 fl1 w96"> <div class="js-post-menu pt2" data-post-id="8456" data-post-type-id="1"> <div class="d-flex gs8 s-anchors s-anchors__muted fw-wrap"> <div class="flex--item"> <a href="/q/8456" rel="nofollow" itemprop="url" class="js-share-link js-gps-track" title="Short permalink to this question" data-gps-track="post.click({ item: 2, priv: 0, post_type: 1 })" data-controller="se-share-sheet" data-se-share-sheet-title="Share a link to this question" data-se-share-sheet-subtitle="" data-se-share-sheet-post-type="question" data-se-share-sheet-social="facebook twitter " data-se-share-sheet-location="1" data-se-share-sheet-license-url="https%3a%2f%2fcreativecommons.org%2flicenses%2fby-sa%2f3.0%2f" data-se-share-sheet-license-name="CC BY-SA 3.0" data-s-popover-placement="bottom-start">Share</a> </div> <div class="flex--item"> <a href="/posts/8456/edit" class="js-suggest-edit-post js-gps-track" data-gps-track="post.click({ item: 6, priv: 0, post_type: 1 })" title="">Improve this question</a> </div> <div class="flex--item"> <button type="button" id="btnFollowPost-8456" class="s-btn s-btn__link js-follow-post js-follow-question js-gps-track" data-gps-track="post.click({ item: 14, priv: 0, post_type: 1 })" data-controller="s-tooltip " data-s-tooltip-placement="bottom" data-s-popover-placement="bottom" aria-controls="" title="Follow this question to receive notifications"> Follow <input type="hidden" id="voteFollowHash" value="66:3:31e,16:b6acc2fd91e83d00,10:1735780817,16:ed07a07d9c628b42,4:8456,1bc3c695ebb997e7d606c9984fc840e5e49c1c3ae01c33277f6e739d444653f6" /> </button> </div> </div> <div class="js-menu-popup-container"></div> </div> </div> <div class="post-signature flex--item"> <div class="user-info "> <div class="d-flex "> <div class="user-action-time fl-grow1"> <a href="/posts/8456/revisions" title="show all edits to this post" class="js-gps-track" data-gps-track="post.click({ item: 4, priv: 0, post_type: 1 })">edited <span title='2015-07-31 20:06:56Z' class='relativetime'>Jul 31, 2015 at 20:06</span></a> </div> </div> <div class="user-gravatar32"> </div> <div class="user-details" itemprop="author" itemscope itemtype="http://schema.org/Person"> <span class="d-none" itemprop="name">MathWanderer</span> <div class="-flair"> </div> </div> </div> </div> <div class="post-signature owner flex--item"> <div class="user-info user-hover "> <div class="d-flex "> <div class="user-action-time fl-grow1"> asked <span title='2015-07-31 19:05:46Z' class='relativetime'>Jul 31, 2015 at 19:05</span> </div> </div> <div class="user-gravatar32"> <a href="/users/5399/mathwanderer"><div class="gravatar-wrapper-32"><img src="https://www.gravatar.com/avatar/1ae3afe7f7393170fb66a6f94dcaf637?s=64&d=identicon&r=PG&f=y&so-version=2" alt="MathWanderer's user avatar" width="32" height="32" class="bar-sm"></div></a> </div> <div class="user-details" itemprop="author" itemscope itemtype="http://schema.org/Person"> <a href="/users/5399/mathwanderer">MathWanderer</a><span class="d-none" itemprop="name">MathWanderer</span> <div class="-flair"> <span class="reputation-score" title="reputation score " dir="ltr">557</span><span title="2 silver badges" aria-hidden="true"><span class="badge2"></span><span class="badgecount">2</span></span><span class="v-visible-sr">2 silver badges</span><span title="8 bronze badges" aria-hidden="true"><span class="badge3"></span><span class="badgecount">8</span></span><span class="v-visible-sr">8 bronze badges</span> </div> </div> </div> </div> </div> </div> <span class="d-none">$\endgroup$</span> </div> <span class="d-none" itemprop="commentCount">0</span> <div class="post-layout--right js-post-comments-component"> <div id="comments-8456" class="comments js-comments-container bt bc-black-200 mt12 dno" data-post-id="8456" data-min-length="15"> <ul class="comments-list js-comments-list" data-remaining-comments-count="0" data-canpost="false" data-cansee="true" data-comments-unavailable="false" data-addlink-disabled="true"> </ul> </div> <div id="comments-link-8456" data-rep=50 data-anon=true> <a class="js-add-link comments-link disabled-link" title="Use comments to ask for more information or suggest improvements. Avoid answering questions in comments." href="#" role="button">Add a comment</a> <span class="js-link-separator dno"> | </span> <a class="js-show-link comments-link dno" title="Expand to show all comments on this post" href=# onclick="" role="button"></a> </div> </div> </div> </div> <div id="answers"> <a name="tab-top"></a> <div id="answers-header"> <div class="answers-subheader d-flex ai-center mb8"> <div class="flex--item fl1"> <h2 class="mb0" data-answercount="4"> 4 Answers <span style="display:none;" itemprop="answerCount">4</span> </h2> </div> <div class="flex--item"> <div class="d-flex g4 gsx ai-center sm:fd-column sm:ai-start"> <div class="d-flex fd-column ai-end sm:ai-start"> <label class="flex--item fs-caption" for="answer-sort-dropdown-select-menu"> Sorted by: </label> <a class="js-sort-preference-change s-link flex--item fs-fine d-none" data-value="ScoreDesc" href="/questions/8456/inquiry-about-my-note-taking-skill?answertab=scoredesc#tab-top" > Reset to default </a> </div> <div class="flex--item s-select"> <select id="answer-sort-dropdown-select-menu"> <option value=scoredesc selected=selected > Highest score (default) </option> <option value=modifieddesc > Date modified (newest first) </option> <option value=createdasc > Date created (oldest first) </option> </select> </div> </div> </div> </div> </div> <a name="8457"></a> <div id="answer-8457" class="answer js-answer" data-answerid="8457" data-parentid="8456" data-score="15" data-position-on-page="1" data-highest-scored="1" data-question-has-accepted-highest-score="0" itemprop="suggestedAnswer" itemscope itemtype="https://schema.org/Answer"> <div class="post-layout"> <div class="votecell post-layout--left"> <div class="js-voting-container d-flex jc-center fd-column ai-center gs4 fc-black-300" data-post-id="8457" data-referrer="None"> <button class="js-vote-up-btn flex--item s-btn s-btn__muted s-btn__outlined bar-pill bc-black-225 f:bc-theme-secondary-400 f:bg-theme-secondary-400 f:fc-black-050 h:bg-theme-primary-200" data-controller="s-tooltip" data-s-tooltip-placement="right" title="This answer is useful" aria-pressed="false" aria-label="Up vote" data-selected-classes="fc-theme-primary-100 bc-theme-primary-500 bg-theme-primary-500" data-unselected-classes="bc-black-225 f:bc-theme-secondary-400 f:bg-theme-secondary-400 f:fc-black-050 h:bg-theme-primary-200"> <svg aria-hidden="true" class="svg-icon iconArrowUp" width="18" height="18" viewBox="0 0 18 18"><path d="M1 12h16L9 4z"/></svg> </button> <input type="hidden" id="voteUpHash" value="66:3:31e,16:ef1059de6a2e2092,10:1735780817,16:ca1de3c6eb5a088e,4:8457,6407a599f0993bdd8ba18e5deec2cce51b1590ecb55bdef23d01ce505e98be9d" /> <div class="js-vote-count flex--item d-flex fd-column ai-center fc-theme-body-font fw-bold fs-subheading py4" itemprop="upvoteCount" data-value="15"> 15 </div> <button class="js-vote-down-btn flex--item mb8 s-btn s-btn__muted s-btn__outlined bar-pill bc-black-225 f:bc-theme-secondary-400 f:bg-theme-secondary-400 f:fc-black-050 h:bg-theme-primary-200" title="This answer is not useful" aria-pressed="false" aria-label="Down vote" data-selected-classes="fc-theme-primary-100 bc-theme-primary-500 bg-theme-primary-500" data-unselected-classes="bc-black-225 f:bc-theme-secondary-400 f:bg-theme-secondary-400 f:fc-black-050 h:bg-theme-primary-200"> <svg aria-hidden="true" class="svg-icon iconArrowDown" width="18" height="18" viewBox="0 0 18 18"><path d="M1 6h16l-8 8z"/></svg> </button> <input type="hidden" id="voteDownHash" value="66:3:31e,16:485b47dbbd04bd86,10:1735780817,16:b77f3f9b844a1e84,4:8457,9ebfa397a4cf430ef74def9394c01c06afbb227ecba57c68adf21168211b2b02" /> <button class="js-saves-btn s-btn s-btn__unset c-pointer py4" type="button" id="saves-btn-8457" data-controller="s-tooltip" data-s-tooltip-placement="right" data-s-popover-placement="" title="Save this answer." data-is-saved="false" aria-label="Save" data-post-id="8457" data-post-type-id="2" data-user-privilege-for-post-click="0" aria-controls="" data-s-popover-auto-show="false" > <svg aria-hidden="true" class="fc-theme-primary-400 js-saves-btn-selected d-none svg-icon iconBookmark" width="18" height="18" viewBox="0 0 18 18"><path d="M3 17V3c0-1.1.9-2 2-2h8a2 2 0 0 1 2 2v14l-6-4z"/></svg> <svg aria-hidden="true" class="js-saves-btn-unselected svg-icon iconBookmarkAlt" width="18" height="18" viewBox="0 0 18 18"><path d="m9 10.6 4 2.66V3H5v10.26zM3 17V3c0-1.1.9-2 2-2h8a2 2 0 0 1 2 2v14l-6-4z"/></svg> </button> <div class="js-accepted-answer-indicator flex--item fc-green-400 py6 mtn8 d-none" data-s-tooltip-placement="right" title="Loading when this answer was accepted…" tabindex="0" role="note" aria-label="Accepted"> <div class="ta-center"> <svg aria-hidden="true" class="svg-icon iconCheckmarkLg" width="36" height="36" viewBox="0 0 36 36"><path d="m6 14 8 8L30 6v8L14 30l-8-8z"/></svg> </div> </div> <a class="js-post-issue flex--item s-btn s-btn__unset c-pointer py6 mx-auto" href="/posts/8457/timeline" data-shortcut="T" data-ks-title="timeline" data-controller="s-tooltip" data-s-tooltip-placement="right" title="Show activity on this post." aria-label="Timeline"><svg aria-hidden="true" class="mln2 mr0 svg-icon iconHistory" width="19" height="18" viewBox="0 0 19 18"><path d="M3 9a8 8 0 1 1 3.73 6.77L8.2 14.3A6 6 0 1 0 5 9l3.01-.01-4 4-4-4zm7-4h1.01L11 9.36l3.22 2.1-.6.93L10 10z"/></svg></a> </div> </div> <div class="answercell post-layout--right"> <span class="d-none">$\begingroup$</span> <div class="s-prose js-post-body" itemprop="text"> <p>As you have noticed, mathematical text is often quite concise and it can be difficult to squeeze it into tighter space or write in your own words in a nontrivial way. Therefore it is easy to end up copying things verbatim if you want to take notes. Taking useful notes in mathematics is quite different from many other subjects, and you are not alone with your problem. You should not be embarrassed; you recognized a problem and want to do something about it and far too few students actively do this. I have some suggestions for taking notes in a more useful way. (This is, of course, based on personal experience and opinion.)</p> <p>Reading a book and following lectures are two different ball games. Let me start with the lectures. It often happens that lectures move on so fast that if you want to copy everything that happens on the blackboard, it can be difficult to follow the content and — most importantly — the ideas. In these days lectures should not be the sole means of delivering the content; the professor can scan his notebook and put the pdf file online for the students to download. If he does not do this, request that he does. Mindless copying is pointless and wastes a lot of time. (Arguing this to some professors can be a difficult task, but it should be tried.) If the lectures are based on slides, ask for them and bring them with you to the lectures, printed on paper. If no such material is available, try to read the textbook or other such material in advance, so that you will have the material in mental form.</p> <p>I have found it very useful to have the lecturer's notes he is using (or very close to it) during the lectures. This does not mean that I sit down idly. I follow the lecture and try to read the material in advance (at least quickly). There are always some tricky points that are not perfectly explained in the written notes. Often the professor says helpful remarks and answers questions from the audience, explaining what is happening or why certain choices were made. It is those gems that I try to catch and write them among the lecture notes. The notes can be informal and do not attempt to make rigorous mathematics. Instead, I try to capture my momentary intuition in words and explain things to myself or relate them to other things. The kinds of notes I take depend heavily on my proficiency in the subject and the notes can be of any kind. (Sometimes I just keep reminding myself about the nature of each object by comments: this is a function, this is a vector, this is a set of sequences…) The notes are rarely so lucid and well explained that no notes are needed, but this can happen, too. Most of the time I just follow the professor's train of thoughts and try to understand why everything happens as it does. Only a small portion of time is spent on making notes. If I have to take notes to copy the content to my notebook (this is sometimes the only way to get it, unfortunately), I have much harder time understanding.</p> <p>If you are reading a book, the situation is different, as there is no hurry. Making supplementary notes works here like during the lectures. If you don't want to damage a book with your own writings, I would suggest post-it notes — I at least give great value to having notes right next to the content they comment. My notes are mostly comments. I find copying theorems and proofs verbatim quite useless. I would suggest reading the theorem and its proof in detail and then trying to figure out what it is about, what it means. Make sure you understand the overall plan of the proof, not just the technical details. If you want to take notes, put the book aside and write the theorem. If you understood the statement and the underlying machinery well, you should be able to write the statement. Then write a proof, at least a sketchy one. If you get stuck, look at the book; it doesn't matter if you have to look every three lines, as long as you really try to do it yourself first. Now you have notes about the theorem, and you have written them down in your own words and as your own thoughts. It might be almost exactly the same proof as in the book, and that's ok. It came from someone else, but it's your proof now.</p> <p>If I have to choose one of your strategies, I would go with 3. Do what feels comfortable and natural to yourself, but make sure there is a point to the notes you are taking.</p> <p>There is one more thing that I must add. Doing exercises is very important in learning mathematics. (I don't know about microbiology.) Do them, and if there are none, make some up and do them. I think the most important notes I have taken in my years of studying mathematics and theoretical physics are solutions to exercises. Exercises give you hand-on experience of the power of your new theorems and make you comfortable with the machinery and language of the theory you are studying.</p> <p>Something of a summary: Take notes only when there is your own thought in them. If you find yourself copying without thinking, there should be another path to the bulk of content. Use notes to explain things to yourself, not to build all of the theory.</p> <p><strong>Addendum:</strong> Let me elaborate on making up your own exercises. (I did this in the comments, but it deserves to be in the actual answer.) If you are curious enough, you don't need to produce problems artificially. Try to dig a little deeper than your course. How are different things related? For example, is the sum of injective functions injective, or is a connected set always measurable, or is every square root of a continuous operator continuous? The questions can be anything, and they can be silly. Is a smooth function a Banach space, or is a finite group measurable? Learning what things are completely unrelated and what things don't even mean anything is useful. I find this kind of mental gymnastics entertaining and it gives me a better touch to the subject.</p> <p>If no questions occur, another good source of questions are the main results of your material. Can you write a list of them without looking? Can you prove the things on your list? Can you define all relevant objects? Do the theorems remain true if you assume a little less? Why not, why yes, or why is it hard to see with the tools you have?</p> <p>In my opinion, informal exercises are important. The problems in exercise sets in books, courses and exams tend to be quite formal, but that doesn't mean that only formal mathematics is good mathematics. Play with your new tools and you'll learn to work with them more efficiently.</p> </div> <div class="mt24"> <div class="d-flex fw-wrap ai-start jc-end gs8 gsy"> <time itemprop="dateCreated" datetime="2015-07-31T22:24:39"></time> <div class="flex--item mr16" style="flex: 1 1 100px;"> <div class="js-post-menu pt2" data-post-id="8457" data-post-type-id="2"> <div class="d-flex gs8 s-anchors s-anchors__muted fw-wrap"> <div class="flex--item"> <a href="/a/8457" rel="nofollow" itemprop="url" class="js-share-link js-gps-track" title="Short permalink to this answer" data-gps-track="post.click({ item: 2, priv: 0, post_type: 2 })" data-controller="se-share-sheet" data-se-share-sheet-title="Share a link to this answer" data-se-share-sheet-subtitle="" data-se-share-sheet-post-type="answer" data-se-share-sheet-social="facebook twitter " data-se-share-sheet-location="2" data-se-share-sheet-license-url="https%3a%2f%2fcreativecommons.org%2flicenses%2fby-sa%2f3.0%2f" data-se-share-sheet-license-name="CC BY-SA 3.0" data-s-popover-placement="bottom-start">Share</a> </div> <div class="flex--item"> <a href="/posts/8457/edit" class="js-suggest-edit-post js-gps-track" data-gps-track="post.click({ item: 6, priv: 0, post_type: 2 })" title="">Improve this answer</a> </div> <div class="flex--item"> <button type="button" id="btnFollowPost-8457" class="s-btn s-btn__link js-follow-post js-follow-answer js-gps-track" data-gps-track="post.click({ item: 14, priv: 0, post_type: 2 })" data-controller="s-tooltip " data-s-tooltip-placement="bottom" data-s-popover-placement="bottom" aria-controls="" title="Follow this answer to receive notifications"> Follow <input type="hidden" id="voteFollowHash" value="66:3:31e,16:9a1ad5dbc5a933de,10:1735780817,16:54c6787a817e2d33,4:8457,6977c043000af94a7ebdc2bd10e6821d86d8874c4535f0fe7d6e9da7236cefee" /> </button> </div> </div> <div class="js-menu-popup-container"></div> </div> </div> <div class="post-signature flex--item fl0"> <div class="user-info "> <div class="d-flex "> <div class="user-action-time fl-grow1"> <a href="/posts/8457/revisions" title="show all edits to this post" class="js-gps-track" data-gps-track="post.click({ item: 4, priv: 0, post_type: 2 })">edited <span title='2016-01-18 14:37:55Z' class='relativetime'>Jan 18, 2016 at 14:37</span></a> </div> </div> <div class="user-gravatar32"> </div> <div class="user-details"> <div class="-flair"> </div> </div> </div> </div> <div class="post-signature flex--item fl0"> <div class="user-info user-hover "> <div class="d-flex "> <div class="user-action-time fl-grow1"> answered <span title='2015-07-31 22:24:39Z' class='relativetime'>Jul 31, 2015 at 22:24</span> </div> </div> <div class="user-gravatar32"> <a href="/users/2074/joonas-ilmavirta"><div class="gravatar-wrapper-32"><img src="https://i.sstatic.net/KPKVZ.png?s=64" alt="Joonas Ilmavirta's user avatar" width="32" height="32" class="bar-sm"></div></a> </div> <div class="user-details" itemprop="author" itemscope itemtype="http://schema.org/Person"> <a href="/users/2074/joonas-ilmavirta">Joonas Ilmavirta</a><span class="d-none" itemprop="name">Joonas Ilmavirta</span> <div class="-flair"> <span class="reputation-score" title="reputation score " dir="ltr">3,770</span><span title="1 gold badge" aria-hidden="true"><span class="badge1"></span><span class="badgecount">1</span></span><span class="v-visible-sr">1 gold badge</span><span title="19 silver badges" aria-hidden="true"><span class="badge2"></span><span class="badgecount">19</span></span><span class="v-visible-sr">19 silver badges</span><span title="51 bronze badges" aria-hidden="true"><span class="badge3"></span><span class="badgecount">51</span></span><span class="v-visible-sr">51 bronze badges</span> </div> </div> </div> </div> </div> </div> <span class="d-none">$\endgroup$</span> </div> <span class="d-none" itemprop="commentCount">8</span> <div class="post-layout--right js-post-comments-component"> <div id="comments-8457" class="comments js-comments-container bt bc-black-200 mt12 " data-post-id="8457" data-min-length="15"> <ul class="comments-list js-comments-list" data-remaining-comments-count="3" data-canpost="false" data-cansee="true" data-comments-unavailable="false" data-addlink-disabled="true"> <li id="comment-18930" class="comment js-comment " data-comment-id="18930" data-comment-owner-id="123" data-comment-score="0"> <div class="js-comment-actions comment-actions"> <div class="comment-score js-comment-score js-comment-edit-hide"> </div> </div> <div class="comment-text js-comment-text-and-form"> <div class="comment-body js-comment-edit-hide"> <span class="d-none">$\begingroup$</span> <span class="comment-copy">I'd add trying to find alternative proofs (or parts of the proof). See if you can extend the result (i.e., it is proved for positive $x$, what about negative ones? Zero?). Poking at the limits helps in understanding why they are there.</span> <span class="d-none">$\endgroup$</span> <div class="d-inline-flex ai-center"> – <a href="/users/123/vonbrand" title="12,464 reputation" class="comment-user">vonbrand</a> </div> <span class="comment-date" dir="ltr"> <span class="v-visible-sr">Commented</span> <span title='2015-08-01 00:38:43Z, License: CC BY-SA 3.0' class='relativetime-clean'>Aug 1, 2015 at 0:38</span> </span> </div> </div> </li> <li id="comment-18941" class="comment js-comment " data-comment-id="18941" data-comment-owner-id="2074" data-comment-score="3"> <div class="js-comment-actions comment-actions"> <div class="comment-score js-comment-score js-comment-edit-hide"> <span title="number of 'useful comment' votes received" class="cool">3</span> </div> </div> <div class="comment-text js-comment-text-and-form"> <div class="comment-body js-comment-edit-hide"> <span class="d-none">$\begingroup$</span> <span class="comment-copy">@MathWanderer, your course of action sounds like a good one. For me little exercises come up naturally when I think about the different objects and what they could do. Are there relations between different properties and operations? A possible question of idle curiosity could ask if the sum of invertible matrices is invertible. You can find counterexamples, but there also seem to be cases when you can guarantee invertibility of the sum... Ask yourself questions, however stupid, and answer them. Try to see structures and places where there is no structure. Dig deeper.</span> <span class="d-none">$\endgroup$</span> <div class="d-inline-flex ai-center"> – <a href="/users/2074/joonas-ilmavirta" title="3,770 reputation" class="comment-user">Joonas Ilmavirta</a> </div> <span class="comment-date" dir="ltr"> <span class="v-visible-sr">Commented</span> <span title='2015-08-01 10:09:43Z, License: CC BY-SA 3.0' class='relativetime-clean'>Aug 1, 2015 at 10:09</span> </span> </div> </div> </li> <li id="comment-18942" class="comment js-comment " data-comment-id="18942" data-comment-owner-id="2074" data-comment-score="1"> <div class="js-comment-actions comment-actions"> <div class="comment-score js-comment-score js-comment-edit-hide"> <span title="number of 'useful comment' votes received" class="cool">1</span> </div> </div> <div class="comment-text js-comment-text-and-form"> <div class="comment-body js-comment-edit-hide"> <span class="d-none">$\begingroup$</span> <span class="comment-copy">@MathWanderer: Questions should stem from curiosity. If you learn a new thing, some questions will surely arise. Ask them. If the teacher or the book can't answer, answer yourself! If you can get old exam questions, solve them. They are often slightly different than exercises during the course. After a while of studying the subject, sit back and think what are the most important objects and theorems about them. Make a big exercise: Define the key objects, write down the key theorems and prove them. Make an overview. I often do this mentally, but writing down is not a bad idea either.</span> <span class="d-none">$\endgroup$</span> <div class="d-inline-flex ai-center"> – <a href="/users/2074/joonas-ilmavirta" title="3,770 reputation" class="comment-user">Joonas Ilmavirta</a> </div> <span class="comment-date" dir="ltr"> <span class="v-visible-sr">Commented</span> <span title='2015-08-01 10:16:45Z, License: CC BY-SA 3.0' class='relativetime-clean'>Aug 1, 2015 at 10:16</span> </span> </div> </div> </li> <li id="comment-18984" class="comment js-comment " data-comment-id="18984" data-comment-owner-id="2074" data-comment-score="2"> <div class="js-comment-actions comment-actions"> <div class="comment-score js-comment-score js-comment-edit-hide"> <span title="number of 'useful comment' votes received" class="cool">2</span> </div> </div> <div class="comment-text js-comment-text-and-form"> <div class="comment-body js-comment-edit-hide"> <span class="d-none">$\begingroup$</span> <span class="comment-copy">@MathWanderer, I added a note at the end of my answer, because I felt that making your own exercises is an important thing and shouldn't be hidden in the comments.</span> <span class="d-none">$\endgroup$</span> <div class="d-inline-flex ai-center"> – <a href="/users/2074/joonas-ilmavirta" title="3,770 reputation" class="comment-user">Joonas Ilmavirta</a> </div> <span class="comment-date" dir="ltr"> <span class="v-visible-sr">Commented</span> <span title='2015-08-05 20:24:14Z, License: CC BY-SA 3.0' class='relativetime-clean'>Aug 5, 2015 at 20:24</span> </span> </div> </div> </li> <li id="comment-18986" class="comment js-comment " data-comment-id="18986" data-comment-owner-id="128" data-comment-score="2"> <div class="js-comment-actions comment-actions"> <div class="comment-score js-comment-score js-comment-edit-hide"> <span title="number of 'useful comment' votes received" class="cool">2</span> </div> </div> <div class="comment-text js-comment-text-and-form"> <div class="comment-body js-comment-edit-hide"> <span class="d-none">$\begingroup$</span> <span class="comment-copy">concerning: "In these days lectures should not be the sole means of delivering the content; the professor can scan his notebook and put the pdf file online for the students to download. If he does not do this, request that he does." I would advise caution on this. There are certainly professors (not me) who would be offended by this. For some, the expectation is that it is your job to take notes and the expectation that they do it for you is lazy. Of course, you are not being lazy, but, still, beware this action has consequences in certain cases.</span> <span class="d-none">$\endgroup$</span> <div class="d-inline-flex ai-center"> – <a href="/users/128/james-s-cook" title="11,040 reputation" class="comment-user">James S. Cook</a> </div> <span class="comment-date" dir="ltr"> <span class="v-visible-sr">Commented</span> <span title='2015-08-06 03:24:27Z, License: CC BY-SA 3.0' class='relativetime-clean'>Aug 6, 2015 at 3:24</span> </span> </div> </div> </li> </ul> </div> <div id="comments-link-8457" data-rep=50 data-anon=true> <a class="js-add-link comments-link dno" title="Use comments to ask for more information or suggest improvements. Avoid comments like “+1” or “thanks”." href="#" role="button"></a> <span class="js-link-separator dno"> | </span> <a class="js-show-link comments-link " title="Expand to show all comments on this post" href=# onclick="" role="button">Show <b>3</b> more comments</a> </div> </div> </div> </div> <div class="js-zone-container zone-container-main"> <div id="dfp-mlb" class="everyonelovesstackoverflow everyoneloves__mid-leaderboard everyoneloves__leaderboard"></div> <div class="js-report-ad-button-container " style="width: 728px"></div> </div> <a name="8475"></a> <div id="answer-8475" class="answer js-answer" data-answerid="8475" data-parentid="8456" data-score="3" data-position-on-page="2" data-highest-scored="0" data-question-has-accepted-highest-score="0" itemprop="suggestedAnswer" itemscope itemtype="https://schema.org/Answer"> <div class="post-layout"> <div class="votecell post-layout--left"> <div class="js-voting-container d-flex jc-center fd-column ai-center gs4 fc-black-300" data-post-id="8475" data-referrer="None"> <button class="js-vote-up-btn flex--item s-btn s-btn__muted s-btn__outlined bar-pill bc-black-225 f:bc-theme-secondary-400 f:bg-theme-secondary-400 f:fc-black-050 h:bg-theme-primary-200" data-controller="s-tooltip" data-s-tooltip-placement="right" title="This answer is useful" aria-pressed="false" aria-label="Up vote" data-selected-classes="fc-theme-primary-100 bc-theme-primary-500 bg-theme-primary-500" data-unselected-classes="bc-black-225 f:bc-theme-secondary-400 f:bg-theme-secondary-400 f:fc-black-050 h:bg-theme-primary-200"> <svg aria-hidden="true" class="svg-icon iconArrowUp" width="18" height="18" viewBox="0 0 18 18"><path d="M1 12h16L9 4z"/></svg> </button> <input type="hidden" id="voteUpHash" value="66:3:31e,16:9f7110603686564f,10:1735780817,16:54190ef17a1661e0,4:8475,c355a5610a2610bd993b5530eb324c74705d857ded7b31d78c498799bb0c05a9" /> <div class="js-vote-count flex--item d-flex fd-column ai-center fc-theme-body-font fw-bold fs-subheading py4" itemprop="upvoteCount" data-value="3"> 3 </div> <button class="js-vote-down-btn flex--item mb8 s-btn s-btn__muted s-btn__outlined bar-pill bc-black-225 f:bc-theme-secondary-400 f:bg-theme-secondary-400 f:fc-black-050 h:bg-theme-primary-200" title="This answer is not useful" aria-pressed="false" aria-label="Down vote" data-selected-classes="fc-theme-primary-100 bc-theme-primary-500 bg-theme-primary-500" data-unselected-classes="bc-black-225 f:bc-theme-secondary-400 f:bg-theme-secondary-400 f:fc-black-050 h:bg-theme-primary-200"> <svg aria-hidden="true" class="svg-icon iconArrowDown" width="18" height="18" viewBox="0 0 18 18"><path d="M1 6h16l-8 8z"/></svg> </button> <input type="hidden" id="voteDownHash" value="66:3:31e,16:cc8168d90fec7d02,10:1735780817,16:9183d1600f481d01,4:8475,16b7c2e9001da3b61a50e43ddc2738b25f07869153cea9e67b0171f493df240c" /> <button class="js-saves-btn s-btn s-btn__unset c-pointer py4" type="button" id="saves-btn-8475" data-controller="s-tooltip" data-s-tooltip-placement="right" data-s-popover-placement="" title="Save this answer." data-is-saved="false" aria-label="Save" data-post-id="8475" data-post-type-id="2" data-user-privilege-for-post-click="0" aria-controls="" data-s-popover-auto-show="false" > <svg aria-hidden="true" class="fc-theme-primary-400 js-saves-btn-selected d-none svg-icon iconBookmark" width="18" height="18" viewBox="0 0 18 18"><path d="M3 17V3c0-1.1.9-2 2-2h8a2 2 0 0 1 2 2v14l-6-4z"/></svg> <svg aria-hidden="true" class="js-saves-btn-unselected svg-icon iconBookmarkAlt" width="18" height="18" viewBox="0 0 18 18"><path d="m9 10.6 4 2.66V3H5v10.26zM3 17V3c0-1.1.9-2 2-2h8a2 2 0 0 1 2 2v14l-6-4z"/></svg> </button> <div class="js-accepted-answer-indicator flex--item fc-green-400 py6 mtn8 d-none" data-s-tooltip-placement="right" title="Loading when this answer was accepted…" tabindex="0" role="note" aria-label="Accepted"> <div class="ta-center"> <svg aria-hidden="true" class="svg-icon iconCheckmarkLg" width="36" height="36" viewBox="0 0 36 36"><path d="m6 14 8 8L30 6v8L14 30l-8-8z"/></svg> </div> </div> <a class="js-post-issue flex--item s-btn s-btn__unset c-pointer py6 mx-auto" href="/posts/8475/timeline" data-shortcut="T" data-ks-title="timeline" data-controller="s-tooltip" data-s-tooltip-placement="right" title="Show activity on this post." aria-label="Timeline"><svg aria-hidden="true" class="mln2 mr0 svg-icon iconHistory" width="19" height="18" viewBox="0 0 19 18"><path d="M3 9a8 8 0 1 1 3.73 6.77L8.2 14.3A6 6 0 1 0 5 9l3.01-.01-4 4-4-4zm7-4h1.01L11 9.36l3.22 2.1-.6.93L10 10z"/></svg></a> </div> </div> <div class="answercell post-layout--right"> <span class="d-none">$\begingroup$</span> <div class="s-prose js-post-body" itemprop="text"> <p>What works for me may not work for you. But here goes with what worked well for me:</p> <ol> <li><p>I used the textbook as my notes. It had an index and was better written (and prettier!) than my handwriting. Where the textbook was too dense or omitted steps in proofs that I needed to make explicit, I added notes in the margins. This sounds like your option #1.</p></li> <li><p>What's more important than "how to take notes" is this: Read the text. Read everything from titles, to captions and footnotes, to the example problems. Don't let ONE WORD get past you. If there is ANYTHNG you don't understand, keep at it until you do understand it or ask for help. This sounds a little like your option #3. And it's not necessarily mutually exclusive with your option #1.</p></li> <li><p>What's more important than ABSORBING the text is working the problems. Even if you completely understand everything in the text, you need the practice--the fluency--that working the problems gives you. In college, I spent about 4 hrs a night 3 nights a week doing calculus homework. But it paid off.</p></li> </ol> </div> <div class="mt24"> <div class="d-flex fw-wrap ai-start jc-end gs8 gsy"> <time itemprop="dateCreated" datetime="2015-08-06T19:57:13"></time> <div class="flex--item mr16" style="flex: 1 1 100px;"> <div class="js-post-menu pt2" data-post-id="8475" data-post-type-id="2"> <div class="d-flex gs8 s-anchors s-anchors__muted fw-wrap"> <div class="flex--item"> <a href="/a/8475" rel="nofollow" itemprop="url" class="js-share-link js-gps-track" title="Short permalink to this answer" data-gps-track="post.click({ item: 2, priv: 0, post_type: 2 })" data-controller="se-share-sheet" data-se-share-sheet-title="Share a link to this answer" data-se-share-sheet-subtitle="" data-se-share-sheet-post-type="answer" data-se-share-sheet-social="facebook twitter " data-se-share-sheet-location="2" data-se-share-sheet-license-url="https%3a%2f%2fcreativecommons.org%2flicenses%2fby-sa%2f3.0%2f" data-se-share-sheet-license-name="CC BY-SA 3.0" data-s-popover-placement="bottom-start">Share</a> </div> <div class="flex--item"> <a href="/posts/8475/edit" class="js-suggest-edit-post js-gps-track" data-gps-track="post.click({ item: 6, priv: 0, post_type: 2 })" title="">Improve this answer</a> </div> <div class="flex--item"> <button type="button" id="btnFollowPost-8475" class="s-btn s-btn__link js-follow-post js-follow-answer js-gps-track" data-gps-track="post.click({ item: 14, priv: 0, post_type: 2 })" data-controller="s-tooltip " data-s-tooltip-placement="bottom" data-s-popover-placement="bottom" aria-controls="" title="Follow this answer to receive notifications"> Follow <input type="hidden" id="voteFollowHash" value="66:3:31e,16:4bfb4bf2957aad4a,10:1735780817,16:25976e2a048ba89c,4:8475,fcbf37e58dc0ba636541a15815e44bb3beb0a7f449d207cca0424e9d9772b48d" /> </button> </div> </div> <div class="js-menu-popup-container"></div> </div> </div> <div class="post-signature flex--item fl0"> <div class="user-info "> <div class="d-flex "> <div class="user-action-time fl-grow1"> answered <span title='2015-08-06 19:57:13Z' class='relativetime'>Aug 6, 2015 at 19:57</span> </div> </div> <div class="user-gravatar32"> <a href="/users/5465/syntax-junkie"><div class="gravatar-wrapper-32"><img src="https://www.gravatar.com/avatar/f911774749a6cad95d07c7f63a01bed3?s=64&d=identicon&r=PG&f=y&so-version=2" alt="Syntax Junkie's user avatar" width="32" height="32" class="bar-sm"></div></a> </div> <div class="user-details" itemprop="author" itemscope itemtype="http://schema.org/Person"> <a href="/users/5465/syntax-junkie">Syntax Junkie</a><span class="d-none" itemprop="name">Syntax Junkie</span> <div class="-flair"> <span class="reputation-score" title="reputation score " dir="ltr">1,386</span><span title="9 silver badges" aria-hidden="true"><span class="badge2"></span><span class="badgecount">9</span></span><span class="v-visible-sr">9 silver badges</span><span title="10 bronze badges" aria-hidden="true"><span class="badge3"></span><span class="badgecount">10</span></span><span class="v-visible-sr">10 bronze badges</span> </div> </div> </div> </div> </div> </div> <span class="d-none">$\endgroup$</span> </div> <span class="d-none" itemprop="commentCount">1</span> <div class="post-layout--right js-post-comments-component"> <div id="comments-8475" class="comments js-comments-container bt bc-black-200 mt12 " data-post-id="8475" data-min-length="15"> <ul class="comments-list js-comments-list" data-remaining-comments-count="0" data-canpost="false" data-cansee="true" data-comments-unavailable="false" data-addlink-disabled="true"> <li id="comment-22865" class="comment js-comment " data-comment-id="22865" data-comment-owner-id="123" data-comment-score="0"> <div class="js-comment-actions comment-actions"> <div class="comment-score js-comment-score js-comment-edit-hide"> </div> </div> <div class="comment-text js-comment-text-and-form"> <div class="comment-body js-comment-edit-hide"> <span class="d-none">$\begingroup$</span> <span class="comment-copy">Don't just do the exercises, work over the derivations, the proofs, and the examples.</span> <span class="d-none">$\endgroup$</span> <div class="d-inline-flex ai-center"> – <a href="/users/123/vonbrand" title="12,464 reputation" class="comment-user">vonbrand</a> </div> <span class="comment-date" dir="ltr"> <span class="v-visible-sr">Commented</span> <span title='2016-01-22 15:08:07Z, License: CC BY-SA 3.0' class='relativetime-clean'>Jan 22, 2016 at 15:08</span> </span> </div> </div> </li> </ul> </div> <div id="comments-link-8475" data-rep=50 data-anon=true> <a class="js-add-link comments-link disabled-link" title="Use comments to ask for more information or suggest improvements. Avoid comments like “+1” or “thanks”." href="#" role="button">Add a comment</a> <span class="js-link-separator dno"> | </span> <a class="js-show-link comments-link dno" title="Expand to show all comments on this post" href=# onclick="" role="button"></a> </div> </div> </div> </div> <a name="10464"></a> <div id="answer-10464" class="answer js-answer" data-answerid="10464" data-parentid="8456" data-score="3" data-position-on-page="3" data-highest-scored="0" data-question-has-accepted-highest-score="0" itemprop="suggestedAnswer" itemscope itemtype="https://schema.org/Answer"> <div class="post-layout"> <div class="votecell post-layout--left"> <div class="js-voting-container d-flex jc-center fd-column ai-center gs4 fc-black-300" data-post-id="10464" data-referrer="None"> <button class="js-vote-up-btn flex--item s-btn s-btn__muted s-btn__outlined bar-pill bc-black-225 f:bc-theme-secondary-400 f:bg-theme-secondary-400 f:fc-black-050 h:bg-theme-primary-200" data-controller="s-tooltip" data-s-tooltip-placement="right" title="This answer is useful" aria-pressed="false" aria-label="Up vote" data-selected-classes="fc-theme-primary-100 bc-theme-primary-500 bg-theme-primary-500" data-unselected-classes="bc-black-225 f:bc-theme-secondary-400 f:bg-theme-secondary-400 f:fc-black-050 h:bg-theme-primary-200"> <svg aria-hidden="true" class="svg-icon iconArrowUp" width="18" height="18" viewBox="0 0 18 18"><path d="M1 12h16L9 4z"/></svg> </button> <input type="hidden" id="voteUpHash" value="67:3:31e,16:aca714d74fbfd268,10:1735780817,16:53bbeb8cdc6f4499,5:10464,5c6290a88331fb4a44d47273c53820d27e8b1ddc5bb09dfa82655e5888c84b79" /> <div class="js-vote-count flex--item d-flex fd-column ai-center fc-theme-body-font fw-bold fs-subheading py4" itemprop="upvoteCount" data-value="3"> 3 </div> <button class="js-vote-down-btn flex--item mb8 s-btn s-btn__muted s-btn__outlined bar-pill bc-black-225 f:bc-theme-secondary-400 f:bg-theme-secondary-400 f:fc-black-050 h:bg-theme-primary-200" title="This answer is not useful" aria-pressed="false" aria-label="Down vote" data-selected-classes="fc-theme-primary-100 bc-theme-primary-500 bg-theme-primary-500" data-unselected-classes="bc-black-225 f:bc-theme-secondary-400 f:bg-theme-secondary-400 f:fc-black-050 h:bg-theme-primary-200"> <svg aria-hidden="true" class="svg-icon iconArrowDown" width="18" height="18" viewBox="0 0 18 18"><path d="M1 6h16l-8 8z"/></svg> </button> <input type="hidden" id="voteDownHash" value="67:3:31e,16:21efa697e3a68d70,10:1735780817,16:a5c0c02469df8047,5:10464,becee6205773dfb2e16ac75e917a4371aab7f8fd838f7a1334e45f4ad60b504a" /> <button class="js-saves-btn s-btn s-btn__unset c-pointer py4" type="button" id="saves-btn-10464" data-controller="s-tooltip" data-s-tooltip-placement="right" data-s-popover-placement="" title="Save this answer." data-is-saved="false" aria-label="Save" data-post-id="10464" data-post-type-id="2" data-user-privilege-for-post-click="0" aria-controls="" data-s-popover-auto-show="false" > <svg aria-hidden="true" class="fc-theme-primary-400 js-saves-btn-selected d-none svg-icon iconBookmark" width="18" height="18" viewBox="0 0 18 18"><path d="M3 17V3c0-1.1.9-2 2-2h8a2 2 0 0 1 2 2v14l-6-4z"/></svg> <svg aria-hidden="true" class="js-saves-btn-unselected svg-icon iconBookmarkAlt" width="18" height="18" viewBox="0 0 18 18"><path d="m9 10.6 4 2.66V3H5v10.26zM3 17V3c0-1.1.9-2 2-2h8a2 2 0 0 1 2 2v14l-6-4z"/></svg> </button> <div class="js-accepted-answer-indicator flex--item fc-green-400 py6 mtn8 d-none" data-s-tooltip-placement="right" title="Loading when this answer was accepted…" tabindex="0" role="note" aria-label="Accepted"> <div class="ta-center"> <svg aria-hidden="true" class="svg-icon iconCheckmarkLg" width="36" height="36" viewBox="0 0 36 36"><path d="m6 14 8 8L30 6v8L14 30l-8-8z"/></svg> </div> </div> <a class="js-post-issue flex--item s-btn s-btn__unset c-pointer py6 mx-auto" href="/posts/10464/timeline" data-shortcut="T" data-ks-title="timeline" data-controller="s-tooltip" data-s-tooltip-placement="right" title="Show activity on this post." aria-label="Timeline"><svg aria-hidden="true" class="mln2 mr0 svg-icon iconHistory" width="19" height="18" viewBox="0 0 19 18"><path d="M3 9a8 8 0 1 1 3.73 6.77L8.2 14.3A6 6 0 1 0 5 9l3.01-.01-4 4-4-4zm7-4h1.01L11 9.36l3.22 2.1-.6.93L10 10z"/></svg></a> </div> </div> <div class="answercell post-layout--right"> <span class="d-none">$\begingroup$</span> <div class="s-prose js-post-body" itemprop="text"> <p>For many courses, note taking is concerned with the organization of data for the purposes of recall. Let's call such organized data "information". In mathematics, the purpose of writing is more, borrowing the language of John Dewey, to marshal resources toward a purpose more specific than recall. Let's call information aimed at a specific purpose "knowledge". Of course, one person's knowledge is another's information. Knowledge is subjective to the kinds of problems or actions an individual is motivated to solve or do (again taking from Dewey, a purpose forms when someone plans their way around an obstacle to satisfying an initial impulse). You can see this in the many mathematical blog posts and textbook treatments of the same basic proofs in mathematics. Writing about these things allows the author to arrange certain facts for ready use for the problems the author is interested in.</p> <p>To risk sounding overly male or violent, this perspective of knowledge as "weaponized information" seems to be a good motivation for mathematical writing. You should have a purpose for your note taking…some problems you (or perhaps the community) want to see solved, or ideas that you want to have a use for, and that the information collected marshals facts aiming to the solution to these problems, or the use of these ideas. The necessary selectivity for good note taking comes from having such purposes, and a sure sign that you don't have them is that you feel like you are copying everything down. (I speak from personal experience!) The thing to do under these circumstances is to ask your own questions in response to the text, and to the instructor, to stimulate purpose. A question is such a seed of an impulse that, since you cannot immediately see an answer, yields a purpose as understood above. Just about any question you genuinely ask will help with this. A good many of the concrete suggestions in the above answer of Joonas will help, as well.</p> </div> <div class="mt24"> <div class="d-flex fw-wrap ai-start jc-end gs8 gsy"> <time itemprop="dateCreated" datetime="2016-01-24T23:59:50"></time> <div class="flex--item mr16" style="flex: 1 1 100px;"> <div class="js-post-menu pt2" data-post-id="10464" data-post-type-id="2"> <div class="d-flex gs8 s-anchors s-anchors__muted fw-wrap"> <div class="flex--item"> <a href="/a/10464" rel="nofollow" itemprop="url" class="js-share-link js-gps-track" title="Short permalink to this answer" data-gps-track="post.click({ item: 2, priv: 0, post_type: 2 })" data-controller="se-share-sheet" data-se-share-sheet-title="Share a link to this answer" data-se-share-sheet-subtitle="" data-se-share-sheet-post-type="answer" data-se-share-sheet-social="facebook twitter " data-se-share-sheet-location="2" data-se-share-sheet-license-url="https%3a%2f%2fcreativecommons.org%2flicenses%2fby-sa%2f3.0%2f" data-se-share-sheet-license-name="CC BY-SA 3.0" data-s-popover-placement="bottom-start">Share</a> </div> <div class="flex--item"> <a href="/posts/10464/edit" class="js-suggest-edit-post js-gps-track" data-gps-track="post.click({ item: 6, priv: 0, post_type: 2 })" title="">Improve this answer</a> </div> <div class="flex--item"> <button type="button" id="btnFollowPost-10464" class="s-btn s-btn__link js-follow-post js-follow-answer js-gps-track" data-gps-track="post.click({ item: 14, priv: 0, post_type: 2 })" data-controller="s-tooltip " data-s-tooltip-placement="bottom" data-s-popover-placement="bottom" aria-controls="" title="Follow this answer to receive notifications"> Follow <input type="hidden" id="voteFollowHash" value="67:3:31e,16:14a4f5a89f273c45,10:1735780817,16:e47a667352bd8009,5:10464,87ca51119d4fb896b88e9baf38edf7d0837ec7a2d905ce43ab42323f4cfdc605" /> </button> </div> </div> <div class="js-menu-popup-container"></div> </div> </div> <div class="post-signature flex--item fl0"> <div class="user-info "> <div class="d-flex "> <div class="user-action-time fl-grow1"> <a href="/posts/10464/revisions" title="show all edits to this post" class="js-gps-track" data-gps-track="post.click({ item: 4, priv: 0, post_type: 2 })">edited <span title='2016-01-25 19:11:36Z' class='relativetime'>Jan 25, 2016 at 19:11</span></a> </div> </div> <div class="user-gravatar32"> </div> <div class="user-details"> <div class="-flair"> </div> </div> </div> </div> <div class="post-signature flex--item fl0"> <div class="user-info user-hover "> <div class="d-flex "> <div class="user-action-time fl-grow1"> answered <span title='2016-01-24 23:59:50Z' class='relativetime'>Jan 24, 2016 at 23:59</span> </div> </div> <div class="user-gravatar32"> <a href="/users/354/jon-bannon"><div class="gravatar-wrapper-32"><img src="https://www.gravatar.com/avatar/f907ea27c55d794ed3ffbb41de525eb0?s=64&d=identicon&r=PG" alt="Jon Bannon's user avatar" width="32" height="32" class="bar-sm"></div></a> </div> <div class="user-details" itemprop="author" itemscope itemtype="http://schema.org/Person"> <a href="/users/354/jon-bannon">Jon Bannon</a><span class="d-none" itemprop="name">Jon Bannon</span> <div class="-flair"> <span class="reputation-score" title="reputation score " dir="ltr">6,213</span><span title="20 silver badges" aria-hidden="true"><span class="badge2"></span><span class="badgecount">20</span></span><span class="v-visible-sr">20 silver badges</span><span title="46 bronze badges" aria-hidden="true"><span class="badge3"></span><span class="badgecount">46</span></span><span class="v-visible-sr">46 bronze badges</span> </div> </div> </div> </div> </div> </div> <span class="d-none">$\endgroup$</span> </div> <span class="d-none" itemprop="commentCount">5</span> <div class="post-layout--right js-post-comments-component"> <div id="comments-10464" class="comments js-comments-container bt bc-black-200 mt12 " data-post-id="10464" data-min-length="15"> <ul class="comments-list js-comments-list" data-remaining-comments-count="0" data-canpost="false" data-cansee="true" data-comments-unavailable="false" data-addlink-disabled="true"> <li id="comment-22887" class="comment js-comment " data-comment-id="22887" data-comment-owner-id="2074" data-comment-score="0"> <div class="js-comment-actions comment-actions"> <div class="comment-score js-comment-score js-comment-edit-hide"> </div> </div> <div class="comment-text js-comment-text-and-form"> <div class="comment-body js-comment-edit-hide"> <span class="d-none">$\begingroup$</span> <span class="comment-copy">While I agree that it can be good to learn math and take notes for a specific purpose, I find assessing usefulness somewhat counterproductive. Reasons: (1) Assessment takes time away from focusing on content. (2) One can learn math just for the fun of it, with no aim to solve specific problems. (3) Interesting applications can call for unexpected theory. The student is not always mature enough to filter information in a way that eventually serves them. [That said, you have a good point. I just wouldn't take it as a main principle when studying.]</span> <span class="d-none">$\endgroup$</span> <div class="d-inline-flex ai-center"> – <a href="/users/2074/joonas-ilmavirta" title="3,770 reputation" class="comment-user">Joonas Ilmavirta</a> </div> <span class="comment-date" dir="ltr"> <span class="v-visible-sr">Commented</span> <span title='2016-01-25 08:42:07Z, License: CC BY-SA 3.0' class='relativetime-clean'>Jan 25, 2016 at 8:42</span> </span> </div> </div> </li> <li id="comment-22888" class="comment js-comment " data-comment-id="22888" data-comment-owner-id="354" data-comment-score="0"> <div class="js-comment-actions comment-actions"> <div class="comment-score js-comment-score js-comment-edit-hide"> </div> </div> <div class="comment-text js-comment-text-and-form"> <div class="comment-body js-comment-edit-hide"> <span class="d-none">$\begingroup$</span> <span class="comment-copy">@Joonas Ilmavirta: It is not necessary that the purposes I refer to have to be specific mathematical problems. Actually, your answer is a more detailed version of mine. You have to do <i>something</i> with the notes you are taking, or why are you taking notes? Why not just keep the textbook? Why reorganize the information? Thanks for the comment, though! I think it may not have been clear that I was not referring to specific mathematical problems.</span> <span class="d-none">$\endgroup$</span> <div class="d-inline-flex ai-center"> – <a href="/users/354/jon-bannon" title="6,213 reputation" class="comment-user">Jon Bannon</a> </div> <span class="comment-date" dir="ltr"> <span class="v-visible-sr">Commented</span> <span title='2016-01-25 11:47:53Z, License: CC BY-SA 3.0' class='relativetime-clean'>Jan 25, 2016 at 11:47</span> </span> </div> </div> </li> <li id="comment-22889" class="comment js-comment " data-comment-id="22889" data-comment-owner-id="354" data-comment-score="0"> <div class="js-comment-actions comment-actions"> <div class="comment-score js-comment-score js-comment-edit-hide"> </div> </div> <div class="comment-text js-comment-text-and-form"> <div class="comment-body js-comment-edit-hide"> <span class="d-none">$\begingroup$</span> <span class="comment-copy">I've made some edits to my answer's language to clarify things a bit.</span> <span class="d-none">$\endgroup$</span> <div class="d-inline-flex ai-center"> – <a href="/users/354/jon-bannon" title="6,213 reputation" class="comment-user">Jon Bannon</a> </div> <span class="comment-date" dir="ltr"> <span class="v-visible-sr">Commented</span> <span title='2016-01-25 12:40:53Z, License: CC BY-SA 3.0' class='relativetime-clean'>Jan 25, 2016 at 12:40</span> </span> </div> </div> </li> <li id="comment-22890" class="comment js-comment " data-comment-id="22890" data-comment-owner-id="2074" data-comment-score="1"> <div class="js-comment-actions comment-actions"> <div class="comment-score js-comment-score js-comment-edit-hide"> <span title="number of 'useful comment' votes received" class="cool">1</span> </div> </div> <div class="comment-text js-comment-text-and-form"> <div class="comment-body js-comment-edit-hide"> <span class="d-none">$\begingroup$</span> <span class="comment-copy">Thanks for the elaboration! My comment came out more harsh than intended, but it's good if it triggered a small added explanation. (For the record, the answer is good and earned my immediate upvote.)</span> <span class="d-none">$\endgroup$</span> <div class="d-inline-flex ai-center"> – <a href="/users/2074/joonas-ilmavirta" title="3,770 reputation" class="comment-user">Joonas Ilmavirta</a> </div> <span class="comment-date" dir="ltr"> <span class="v-visible-sr">Commented</span> <span title='2016-01-25 14:12:36Z, License: CC BY-SA 3.0' class='relativetime-clean'>Jan 25, 2016 at 14:12</span> </span> </div> </div> </li> <li id="comment-22904" class="comment js-comment " data-comment-id="22904" data-comment-owner-id="354" data-comment-score="1"> <div class="js-comment-actions comment-actions"> <div class="comment-score js-comment-score js-comment-edit-hide"> <span title="number of 'useful comment' votes received" class="cool">1</span> </div> </div> <div class="comment-text js-comment-text-and-form"> <div class="comment-body js-comment-edit-hide"> <span class="d-none">$\begingroup$</span> <span class="comment-copy">@Joonas Ilmaravirta: I think your (3) above is important to address, so I will, at risk of lengthening this thread a bit too much. While I agree that interesting applications can call for an unexpected theory, one can use up a LOT of energy and time collecting facts with the risk of never gaining any deep proficiency or the ability to evaluate when to apply an idea. Somehow the point of mathematical training should be to develop the ability to filter information as above. If the study purpose is to "look smart for friends", that's fine, but we owe it to students to state community expectations</span> <span class="d-none">$\endgroup$</span> <div class="d-inline-flex ai-center"> – <a href="/users/354/jon-bannon" title="6,213 reputation" class="comment-user">Jon Bannon</a> </div> <span class="comment-date" dir="ltr"> <span class="v-visible-sr">Commented</span> <span title='2016-01-26 15:03:39Z, License: CC BY-SA 3.0' class='relativetime-clean'>Jan 26, 2016 at 15:03</span> </span> </div> </div> </li> </ul> </div> <div id="comments-link-10464" data-rep=50 data-anon=true> <a class="js-add-link comments-link disabled-link" title="Use comments to ask for more information or suggest improvements. Avoid comments like “+1” or “thanks”." href="#" role="button">Add a comment</a> <span class="js-link-separator dno"> | </span> <a class="js-show-link comments-link dno" title="Expand to show all comments on this post" href=# onclick="" role="button"></a> </div> </div> </div> </div> <div class="js-zone-container zone-container-main"> <div id="dfp-smlb" class="everyonelovesstackoverflow everyoneloves__mid-second-leaderboard everyoneloves__leaderboard"></div> <div class="js-report-ad-button-container " style="width: 728px"></div> </div> <a name="8463"></a> <div id="answer-8463" class="answer js-answer" data-answerid="8463" data-parentid="8456" data-score="1" data-position-on-page="4" data-highest-scored="0" data-question-has-accepted-highest-score="0" itemprop="suggestedAnswer" itemscope itemtype="https://schema.org/Answer"> <div class="post-layout"> <div class="votecell post-layout--left"> <div class="js-voting-container d-flex jc-center fd-column ai-center gs4 fc-black-300" data-post-id="8463" data-referrer="None"> <button class="js-vote-up-btn flex--item s-btn s-btn__muted s-btn__outlined bar-pill bc-black-225 f:bc-theme-secondary-400 f:bg-theme-secondary-400 f:fc-black-050 h:bg-theme-primary-200" data-controller="s-tooltip" data-s-tooltip-placement="right" title="This answer is useful" aria-pressed="false" aria-label="Up vote" data-selected-classes="fc-theme-primary-100 bc-theme-primary-500 bg-theme-primary-500" data-unselected-classes="bc-black-225 f:bc-theme-secondary-400 f:bg-theme-secondary-400 f:fc-black-050 h:bg-theme-primary-200"> <svg aria-hidden="true" class="svg-icon iconArrowUp" width="18" height="18" viewBox="0 0 18 18"><path d="M1 12h16L9 4z"/></svg> </button> <input type="hidden" id="voteUpHash" value="66:3:31e,16:2a4c8cce92b30a27,10:1735780817,16:72db714d0acbf6d5,4:8463,ec22bdfdbc1cdec28edc4978755c04deb5fa1dee03d27e4e622ed9449f6d5b3a" /> <div class="js-vote-count flex--item d-flex fd-column ai-center fc-theme-body-font fw-bold fs-subheading py4" itemprop="upvoteCount" data-value="1"> 1 </div> <button class="js-vote-down-btn flex--item mb8 s-btn s-btn__muted s-btn__outlined bar-pill bc-black-225 f:bc-theme-secondary-400 f:bg-theme-secondary-400 f:fc-black-050 h:bg-theme-primary-200" title="This answer is not useful" aria-pressed="false" aria-label="Down vote" data-selected-classes="fc-theme-primary-100 bc-theme-primary-500 bg-theme-primary-500" data-unselected-classes="bc-black-225 f:bc-theme-secondary-400 f:bg-theme-secondary-400 f:fc-black-050 h:bg-theme-primary-200"> <svg aria-hidden="true" class="svg-icon iconArrowDown" width="18" height="18" viewBox="0 0 18 18"><path d="M1 6h16l-8 8z"/></svg> </button> <input type="hidden" id="voteDownHash" value="66:3:31e,16:0d41889c29b0e6a8,10:1735780817,16:082ae0d290421561,4:8463,159a1fa0a19844ed674bf3c10a8c58cf8f71e4b435234a838035230dd3db1024" /> <button class="js-saves-btn s-btn s-btn__unset c-pointer py4" type="button" id="saves-btn-8463" data-controller="s-tooltip" data-s-tooltip-placement="right" data-s-popover-placement="" title="Save this answer." data-is-saved="false" aria-label="Save" data-post-id="8463" data-post-type-id="2" data-user-privilege-for-post-click="0" aria-controls="" data-s-popover-auto-show="false" > <svg aria-hidden="true" class="fc-theme-primary-400 js-saves-btn-selected d-none svg-icon iconBookmark" width="18" height="18" viewBox="0 0 18 18"><path d="M3 17V3c0-1.1.9-2 2-2h8a2 2 0 0 1 2 2v14l-6-4z"/></svg> <svg aria-hidden="true" class="js-saves-btn-unselected svg-icon iconBookmarkAlt" width="18" height="18" viewBox="0 0 18 18"><path d="m9 10.6 4 2.66V3H5v10.26zM3 17V3c0-1.1.9-2 2-2h8a2 2 0 0 1 2 2v14l-6-4z"/></svg> </button> <div class="js-accepted-answer-indicator flex--item fc-green-400 py6 mtn8 d-none" data-s-tooltip-placement="right" title="Loading when this answer was accepted…" tabindex="0" role="note" aria-label="Accepted"> <div class="ta-center"> <svg aria-hidden="true" class="svg-icon iconCheckmarkLg" width="36" height="36" viewBox="0 0 36 36"><path d="m6 14 8 8L30 6v8L14 30l-8-8z"/></svg> </div> </div> <a class="js-post-issue flex--item s-btn s-btn__unset c-pointer py6 mx-auto" href="/posts/8463/timeline" data-shortcut="T" data-ks-title="timeline" data-controller="s-tooltip" data-s-tooltip-placement="right" title="Show activity on this post." aria-label="Timeline"><svg aria-hidden="true" class="mln2 mr0 svg-icon iconHistory" width="19" height="18" viewBox="0 0 19 18"><path d="M3 9a8 8 0 1 1 3.73 6.77L8.2 14.3A6 6 0 1 0 5 9l3.01-.01-4 4-4-4zm7-4h1.01L11 9.36l3.22 2.1-.6.93L10 10z"/></svg></a> </div> </div> <div class="answercell post-layout--right"> <span class="d-none">$\begingroup$</span> <div class="s-prose js-post-body" itemprop="text"> <p>When in lectures, as mentioned above, if you have access to the slides or whatever, that is great! I'd suggest bringing a tape recorder with you, and then when you take notes, either take them on the slides itself, or in your notes write</p> <p>Slide 1: lsdkfjslkfjslkfjsldkfjsljfsdlkfs</p> <p>Slide 2: sdlkfjsldfkjslfkj</p> <p>and only take what you feel at the time is relevant. If you feel you missed something later, you have the voice recording (as you get better at taking notes, and you feel more confident, you can skip the voice recording)</p> <p>You can do the same with your own thoughts when reading a book, and take the page numbers, or since you mentioned that you learn best by reading, read with a highlighter as it may help keep you focused. </p> <p>Creating your own problems can definitely be useful - if you find it hard to do that first, then google whatever type of problems you are struggling with and find something with solutions that you can check after. There are a ton of such problems up on the internet from various courses. </p> <p>As a side note: Most universities have some sort of learning center, study skills center to help students learn various study skills. My suggestion is that you go and take a course on note taking, but also on one that could target your learning style. Although you seem to know your learning style, such courses also usually give you tips on how to best learn, and study etc, keeping your learning style in mind. </p> </div> <div class="mt24"> <div class="d-flex fw-wrap ai-start jc-end gs8 gsy"> <time itemprop="dateCreated" datetime="2015-08-02T09:18:15"></time> <div class="flex--item mr16" style="flex: 1 1 100px;"> <div class="js-post-menu pt2" data-post-id="8463" data-post-type-id="2"> <div class="d-flex gs8 s-anchors s-anchors__muted fw-wrap"> <div class="flex--item"> <a href="/a/8463" rel="nofollow" itemprop="url" class="js-share-link js-gps-track" title="Short permalink to this answer" data-gps-track="post.click({ item: 2, priv: 0, post_type: 2 })" data-controller="se-share-sheet" data-se-share-sheet-title="Share a link to this answer" data-se-share-sheet-subtitle="" data-se-share-sheet-post-type="answer" data-se-share-sheet-social="facebook twitter " data-se-share-sheet-location="2" data-se-share-sheet-license-url="https%3a%2f%2fcreativecommons.org%2flicenses%2fby-sa%2f3.0%2f" data-se-share-sheet-license-name="CC BY-SA 3.0" data-s-popover-placement="bottom-start">Share</a> </div> <div class="flex--item"> <a href="/posts/8463/edit" class="js-suggest-edit-post js-gps-track" data-gps-track="post.click({ item: 6, priv: 0, post_type: 2 })" title="">Improve this answer</a> </div> <div class="flex--item"> <button type="button" id="btnFollowPost-8463" class="s-btn s-btn__link js-follow-post js-follow-answer js-gps-track" data-gps-track="post.click({ item: 14, priv: 0, post_type: 2 })" data-controller="s-tooltip " data-s-tooltip-placement="bottom" data-s-popover-placement="bottom" aria-controls="" title="Follow this answer to receive notifications"> Follow <input type="hidden" id="voteFollowHash" value="66:3:31e,16:cc0ea80846041d53,10:1735780817,16:765b2ac46bc58d22,4:8463,5122046572804c3aec478b94dfebf0aa83c183af2cb173998dc147e2d3ccbf76" /> </button> </div> </div> <div class="js-menu-popup-container"></div> </div> </div> <div class="post-signature flex--item fl0"> <div class="user-info "> <div class="d-flex "> <div class="user-action-time fl-grow1"> answered <span title='2015-08-02 09:18:15Z' class='relativetime'>Aug 2, 2015 at 9:18</span> </div> </div> <div class="user-gravatar32"> <a href="/users/5451/rebecca"><div class="gravatar-wrapper-32"><img src="https://www.gravatar.com/avatar/2889029fa262268785b0c34d58d17533?s=64&d=identicon&r=PG&f=y&so-version=2" alt="Rebecca's user avatar" width="32" height="32" class="bar-sm"></div></a> </div> <div class="user-details" itemprop="author" itemscope itemtype="http://schema.org/Person"> <a href="/users/5451/rebecca">Rebecca</a><span class="d-none" itemprop="name">Rebecca</span> <div class="-flair"> <span class="reputation-score" title="reputation score " dir="ltr">191</span><span title="4 bronze badges" aria-hidden="true"><span class="badge3"></span><span class="badgecount">4</span></span><span class="v-visible-sr">4 bronze badges</span> </div> </div> </div> </div> </div> </div> <span class="d-none">$\endgroup$</span> </div> <span class="d-none" itemprop="commentCount">1</span> <div class="post-layout--right js-post-comments-component"> <div id="comments-8463" class="comments js-comments-container bt bc-black-200 mt12 " data-post-id="8463" data-min-length="15"> <ul class="comments-list js-comments-list" data-remaining-comments-count="0" data-canpost="false" data-cansee="true" data-comments-unavailable="false" data-addlink-disabled="true"> <li id="comment-18985" class="comment js-comment " data-comment-id="18985" data-comment-owner-id="262" data-comment-score="5"> <div class="js-comment-actions comment-actions"> <div class="comment-score js-comment-score js-comment-edit-hide"> <span title="number of 'useful comment' votes received" class="warm">5</span> </div> </div> <div class="comment-text js-comment-text-and-form"> <div class="comment-body js-comment-edit-hide"> <span class="d-none">$\begingroup$</span> <span class="comment-copy"><b>As a side-note:</b> It is probably best to ask the instructor beforehand if it is okay to use a recorder (video or audio) in class. Somewhat related is <a href="http://academia.stackexchange.com/questions/48940"><b>ASE 48940</b></a>.</span> <span class="d-none">$\endgroup$</span> <div class="d-inline-flex ai-center"> – <a href="/users/262/benjamin-dickman" title="18,704 reputation" class="comment-user">Benjamin Dickman</a> </div> <span class="comment-date" dir="ltr"> <span class="v-visible-sr">Commented</span> <span title='2015-08-05 23:23:45Z, License: CC BY-SA 3.0' class='relativetime-clean'>Aug 5, 2015 at 23:23</span> </span> </div> </div> </li> </ul> </div> <div id="comments-link-8463" data-rep=50 data-anon=true> <a class="js-add-link comments-link disabled-link" title="Use comments to ask for more information or suggest improvements. Avoid comments like “+1” or “thanks”." href="#" role="button">Add a comment</a> <span class="js-link-separator dno"> | </span> <a class="js-show-link comments-link dno" title="Expand to show all comments on this post" href=# onclick="" role="button"></a> </div> </div> </div> </div> <a name='new-answer'></a> <form id="post-form" action="/questions/8456/answer/submit" method="post" class="js-add-answer-component post-form"> <input type="hidden" id="post-id" value="8456" /> <input type="hidden" id="qualityBanWarningShown" name="qualityBanWarningShown" value="false" /> <input type="hidden" name="referrer" value="" /> <h2 class="space" id="your-answer-header"> Your Answer </h2> <script> StackExchange.ready(function() { var channelOptions = { tags: "".split(" "), id: "548" }; initTagRenderer("".split(" "), "".split(" "), channelOptions); StackExchange.using("externalEditor", function() { // Have to fire editor after snippets, if snippets enabled if (StackExchange.settings.snippets.snippetsEnabled) { StackExchange.using("snippets", function() { createEditor(); }); } else { createEditor(); } }); function createEditor() { StackExchange.prepareEditor({ useStacksEditor: false, heartbeatType: 'answer', autoActivateHeartbeat: false, convertImagesToLinks: false, noModals: true, showLowRepImageUploadWarning: true, reputationToPostImages: null, bindNavPrevention: true, postfix: "", imageUploadEnabled: false, imageUploader: { brandingHtml: "", contentPolicyHtml: "User contributions licensed under \u003ca href=\"https://stackoverflow.com/help/licensing\"\u003eCC BY-SA\u003c/a\u003e \u003ca href=\"https://stackoverflow.com/legal/acceptable-use-policy\"\u003e(content policy)\u003c/a\u003e", allowUrls: true, }, noCode: true, onDemand: true, discardSelector: ".discard-answer", enableTables: true, isStacksEditorPreviewEnabled: false ,immediatelyShowMarkdownHelp:true,enableTables:true }); } }); </script> <div id="post-editor" class="post-editor js-post-editor d-flex fd-column g4"> <div class="ps-relative"> <div class="wmd-container mb8"> <div id="wmd-button-bar" class="wmd-button-bar btr-sm"></div> <div class="js-stacks-validation"> <div class="ps-relative"> <textarea id="wmd-input" name="post-text" class="wmd-input s-input bar0 js-post-body-field" data-editor-type="wmd" data-post-type-id="2" cols="92" rows="15" aria-labelledby="your-answer-header" tabindex="101" data-min-length=""></textarea> </div> <div class="s-input-message mt4 d-none js-stacks-validation-message"></div> </div> </div> </div> <aside class="d-flex ai-start jc-space-between js-answer-help s-notice s-notice__warning pb0 pr4 pt4 mb8 d-none" role="status" aria-hidden="true"> <div class="flex--item pt8"> <p>Thanks for contributing an answer to Mathematics Educators Stack Exchange!</p><ul><li>Please be sure to <em>answer the question</em>. Provide details and share your research!</li></ul><p>But <em>avoid</em> …</p><ul><li>Asking for help, clarification, or responding to other answers.</li><li>Making statements based on opinion; back them up with references or personal experience.</li></ul><p>Use MathJax to format equations. <a href="https://legacy-www.math.harvard.edu/texman/">MathJax reference</a>.</p><p>To learn more, see our <a href="/help/how-to-answer">tips on writing great answers</a>.</p> </div> <button class="flex--item js-answer-help-close-btn s-btn s-btn__muted fc-black-600"> <svg aria-hidden="true" class="svg-icon iconClear" width="18" height="18" viewBox="0 0 18 18"><path d="M15 4.41 13.59 3 9 7.59 4.41 3 3 4.41 7.59 9 3 13.59 4.41 15 9 10.41 13.59 15 15 13.59 10.41 9z"/></svg> </button> </aside> <div> <div id="draft-saved" class="fc-success h24" style="display:none;">Draft saved</div> <div id="draft-discarded" class="fc-error h24" style="display:none;">Draft discarded</div> </div> <div id="wmd-preview" class="s-prose mb16 wmd-preview js-wmd-preview"></div> <div></div> <div class="edit-block"> <input id="fkey" name="fkey" type="hidden" value="b39cb1a761f93912de068f8e6267ee5702b7d597a570755c5112354b099addd4"> <input id="author" name="author" type="text"> </div> </div> <div class="ps-relative"> <div class="form-item dno new-post-login p0 my16"> <div class="d-flex gs16 md:fd-column new-login-form"> <div class="d-flex fd-column w50 md:w-auto gsy gs8 jc-space-between new-login-left"> <h3 class="flex--item fs-title">Sign up or <a id="login-link" href="/users/login?ssrc=question_page&returnurl=https%3a%2f%2fmatheducators.stackexchange.com%2fquestions%2f8456%2finquiry-about-my-note-taking-skill%23new-answer">log in</a></h3> <script> StackExchange.ready(function () { StackExchange.helpers.onClickDraftSave('#login-link'); }); </script> <div class="flex--item s-btn s-btn__muted s-btn__outlined s-btn__icon google-login" data-ga="["sign up","Sign Up Started - Google","New Post",null,null]"> <svg aria-hidden="true" class="native svg-icon iconGoogle" width="18" height="18" viewBox="0 0 18 18"><path fill="#4285F4" d="M16.51 8H8.98v3h4.3c-.18 1-.74 1.48-1.6 2.04v2.01h2.6a7.8 7.8 0 0 0 2.38-5.88c0-.57-.05-.66-.15-1.18"/><path fill="#34A853" d="M8.98 17c2.16 0 3.97-.72 5.3-1.94l-2.6-2a4.8 4.8 0 0 1-7.18-2.54H1.83v2.07A8 8 0 0 0 8.98 17"/><path fill="#FBBC05" d="M4.5 10.52a4.8 4.8 0 0 1 0-3.04V5.41H1.83a8 8 0 0 0 0 7.18z"/><path fill="#EA4335" d="M8.98 4.18c1.17 0 2.23.4 3.06 1.2l2.3-2.3A8 8 0 0 0 1.83 5.4L4.5 7.49a4.8 4.8 0 0 1 4.48-3.3"/></svg> Sign up using Google </div> <div class="flex--item s-btn s-btn__muted s-btn__outlined s-btn__icon stackexchange-login" data-ga="["sign up","Sign Up Navigation","New Post",null,null]"> <svg aria-hidden="true" class="native svg-icon iconGlyphXSm" width="18" height="18" viewBox="0 0 18 18"><path fill="#BCBBBB" d="M14 16v-5h2v7H2v-7h2v5z"/><path fill="#F48024" d="m12.09.72-1.21.9 4.5 6.07 1.22-.9zM5 15h8v-2H5zm9.15-5.87L8.35 4.3l.96-1.16 5.8 4.83zm-7.7-1.47 6.85 3.19.63-1.37-6.85-3.2zm6.53 5L5.4 11.39l.38-1.67 7.42 1.48z"/></svg> Sign up using Email and Password </div> </div> <input type="hidden" name="use-facebook" class="use-facebook" value="false" /> <input type="hidden" name="use-google" class="use-google" value="false" /> <button type="button" class="d-none js-submit-openid">Submit</button> <div class="d-flex gsy gs8 fd-column w50 md:w-auto new-login-right form-item p0"> <h3 class="flex--item fs-title">Post as a guest</h3> <div class="flex--item"> <div class="d-flex gs4 gsy fd-column"> <label class="s-label" for="display-name">Name</label> <div class="d-flex ps-relative"> <input class="s-input" id="display-name" name="display-name" maxlength="30" type="text" value="" tabindex="105" placeholder="" /> </div> </div> </div> <div class="flex--item"> <div class="d-flex gs4 gsy fd-column"> <div class="flex--item"> <div class="d-flex gs2 gsy fd-column"> <label class="flex--item s-label" for="m-address">Email</label> <p class="flex--item s-description">Required, but never shown</p> </div> </div> <div class="d-flex ps-relative"> <input class="s-input js-post-email-field" id="m-address" name="m-address" type="text" value="" size="40" tabindex="106" placeholder="" /> </div> </div> </div> </div> </div> </div> <script> StackExchange.ready( function () { StackExchange.openid.initPostLogin('.new-post-login', 'https%3a%2f%2fmatheducators.stackexchange.com%2fquestions%2f8456%2finquiry-about-my-note-taking-skill%23new-answer', 'question_page'); } ); </script> <noscript> <h3 class="flex--item fs-title">Post as a guest</h3> <div class="flex--item"> <div class="d-flex gs4 gsy fd-column"> <label class="s-label" for="display-name">Name</label> <div class="d-flex ps-relative"> <input class="s-input" id="display-name" name="display-name" maxlength="30" type="text" value="" tabindex="105" placeholder="" /> </div> </div> </div> <div class="flex--item"> <div class="d-flex gs4 gsy fd-column"> <div class="flex--item"> <div class="d-flex gs2 gsy fd-column"> <label class="flex--item s-label" for="m-address">Email</label> <p class="flex--item s-description">Required, but never shown</p> </div> </div> <div class="d-flex ps-relative"> <input class="s-input js-post-email-field" id="m-address" name="m-address" type="text" value="" size="40" tabindex="106" placeholder="" /> </div> </div> </div> </noscript> </div> <div class="form-submit clear-both d-flex sm:fd-column sm:jc-stretch gs4 ai-center"> <button id="submit-button" class="flex--item fl-shrink0 s-btn s-btn__filled sm:w100" type="submit" tabindex="120" autocomplete="off"> Post Your Answer </button> <button class="flex--item s-btn s-btn__danger fl-shrink0 sm:w100 discard-answer d-none"> Discard </button> <p class="flex--item mb0 fs-italic ml12 sm:ml0"> By clicking “Post Your Answer”, you agree to our <a href='https://stackoverflow.com/legal/terms-of-service/public' name='tos' target='_blank' class='-link'>terms of service</a> and acknowledge you have read our <a href='https://stackoverflow.com/legal/privacy-policy' name='privacy' target='_blank' class='-link'>privacy policy</a>.<input type="hidden" name="legalLinksShown" value="1" /> </p> </div> <div class="js-general-error general-error clear-both d-none" aria-live="polite"></div> </form> <h2 class="bottom-notice" data-loc="1"> <div> Not the answer you're looking for? Browse other questions tagged <ul class='ml0 list-ls-none js-post-tag-list-wrapper d-inline'><li class='d-inline mr4 js-post-tag-list-item'><a href="/questions/tagged/undergraduate-education" class="s-tag post-tag" title="show questions tagged 'undergraduate-education'" aria-label="show questions tagged 'undergraduate-education'" rel="tag" aria-labelledby="tag-undergraduate-education-tooltip-container" data-tag-menu-origin="Unknown">undergraduate-education</a></li><li class='d-inline mr4 js-post-tag-list-item'><a href="/questions/tagged/self-learning" class="s-tag post-tag" title="show questions tagged 'self-learning'" aria-label="show questions tagged 'self-learning'" rel="tag" aria-labelledby="tag-self-learning-tooltip-container" data-tag-menu-origin="Unknown">self-learning</a></li><li class='d-inline mr4 js-post-tag-list-item'><a href="/questions/tagged/lecture-notes" class="s-tag post-tag" title="show questions tagged 'lecture-notes'" aria-label="show questions tagged 'lecture-notes'" rel="tag" aria-labelledby="tag-lecture-notes-tooltip-container" data-tag-menu-origin="Unknown">lecture-notes</a></li></ul> or <a href="/questions/ask">ask your own question</a>. </div> </h2> </div> </div> <div id="sidebar" class="show-votes" role="complementary" aria-label="sidebar"> <div class="s-sidebarwidget s-sidebarwidget__yellow s-anchors s-anchors__grayscale mb16" data-tracker="cb=1"> <ul class="s-sidebarwidget--content s-sidebarwidget__items p0"> <li class="s-sidebarwidget--header"> Featured on Meta </li> <li class="s-sidebarwidget--item d-flex px16"> <div class="flex--item1 fl-shrink0"> <div class="favicon favicon-stackexchangemeta" title="Meta Stack Exchange"></div> </div> <div class="flex--item wmn0 ow-break-word"> <a href="https://meta.stackexchange.com/questions/404909/stack-overflow-jobs-is-expanding-to-more-countries" class="js-gps-track" data-ga="["community bulletin board","Featured on Meta","https://meta.stackexchange.com/questions/404909/stack-overflow-jobs-is-expanding-to-more-countries",null,null]" data-gps-track="communitybulletin.click({ priority: 3, position: 0, location: questionpage })">Stack Overflow Jobs is expanding to more countries</a> </div> </li> <li class="s-sidebarwidget--item d-flex px16"> <div class="flex--item1 fl-shrink0"> <div class="favicon favicon-stackexchangemeta" title="Meta Stack Exchange"></div> </div> <div class="flex--item wmn0 ow-break-word"> <a href="https://meta.stackexchange.com/questions/404724/the-december-2024-community-asks-sprint-has-been-moved-to-march-2025-and-length" class="js-gps-track" title="The December 2024 Community Asks Sprint has been moved to March 2025 (and lengthened to 2 weeks to compensate)" data-ga="["community bulletin board","Featured on Meta","https://meta.stackexchange.com/questions/404724/the-december-2024-community-asks-sprint-has-been-moved-to-march-2025-and-length",null,null]" data-gps-track="communitybulletin.click({ priority: 3, position: 1, location: questionpage })">The December 2024 Community Asks Sprint has been moved to March 2025 (and...</a> </div> </li> </ul> </div> <div class="js-zone-container zone-container-sidebar"> <div id="dfp-tsb" class="everyonelovesstackoverflow everyoneloves__top-sidebar"></div> <div class="js-report-ad-button-container " style="width: 300px"></div> </div> <div class="module sidebar-related"> <h4 id="h-related">Related</h4> <div class="related js-gps-related-questions" data-tracker="rq=1"> <div class="spacer" data-question-id="5736"> <a href="/q/5736" title="Question score (upvotes - downvotes)" > <div class="answer-votes default">6</div> </a> <a href="/questions/5736/how-can-i-make-my-notes-more-useful-for-future-reference" class="question-hyperlink">How can I make my notes more useful for future reference?</a> </div> <div class="spacer" data-question-id="8407"> <a href="/q/8407" title="Question score (upvotes - downvotes)" > <div class="answer-votes default">5</div> </a> <a href="/questions/8407/inquiry-about-my-self-study-plan-for-real-analysis-for-my-research-and-self-enr" class="question-hyperlink">Inquiry about My Self-Study Plan for Real Analysis (for my research and self-enrichment)</a> </div> <div class="spacer" data-question-id="10742"> <a href="/q/10742" title="Question score (upvotes - downvotes)" > <div class="answer-votes answered-accepted default">2</div> </a> <a href="/questions/10742/how-to-take-notes-when-teacher-uses-slides" class="question-hyperlink">How to take notes when teacher uses slides?</a> </div> <div class="spacer" data-question-id="10977"> <a href="/q/10977" title="Question score (upvotes - downvotes)" > <div class="answer-votes default">15</div> </a> <a href="/questions/10977/is-it-better-to-provide-students-with-guided-notes-or-to-have-them-write-their-o" class="question-hyperlink">Is it better to provide students with guided notes or to have them write their own notes, or both?</a> </div> <div class="spacer" data-question-id="14218"> <a href="/q/14218" title="Question score (upvotes - downvotes)" > <div class="answer-votes default">8</div> </a> <a href="/questions/14218/how-to-deal-with-poor-students-who-dont-take-notes" class="question-hyperlink">How to deal with poor students who don't take notes?</a> </div> <div class="spacer" data-question-id="15403"> <a href="/q/15403" title="Question score (upvotes - downvotes)" > <div class="answer-votes answered-accepted default">7</div> </a> <a href="/questions/15403/collaborative-note-taking" class="question-hyperlink">Collaborative note taking</a> </div> <div class="spacer" data-question-id="17157"> <a href="/q/17157" title="Question score (upvotes - downvotes)" > <div class="answer-votes default">5</div> </a> <a href="/questions/17157/is-there-a-rigrous-and-relatively-complete-book-lecture-notes-for-selfstudying-s" class="question-hyperlink">Is there a rigrous and relatively complete book/lecture notes for selfstudying several variable calculus?</a> </div> <div class="spacer" data-question-id="26641"> <a href="/q/26641" title="Question score (upvotes - downvotes)" > <div class="answer-votes answered-accepted default">8</div> </a> <a href="/questions/26641/should-i-really-just-shut-up-and-calculate-on-learning-at-a-good-pace-without" class="question-hyperlink">Should I really just "shut up and calculate"? On learning at a good pace without sacrificing rigour</a> </div> </div> </div> <script type="text/javascript"> $(function() { $(".js-gps-related-questions .spacer").on("click", function () { fireRelatedEvent($(this).index() + 1, $(this).data('question-id')); }); function fireRelatedEvent(position, questionId) { StackExchange.using("gps", function() { StackExchange.gps.track('related_questions.click', { position: position, originQuestionId: 8456, relatedQuestionId: +questionId, location: 'sidebar', source: 'Baseline' }); }); } }); </script> <div id="hot-network-questions" class="module tex2jax_ignore"> <h4> <a href="https://stackexchange.com/questions?tab=hot" class="js-gps-track s-link s-link__inherit" data-gps-track="posts_hot_network.click({ item_type:1, location:11 })"> Hot Network Questions </a> </h4> <ul> <li > <div class="favicon favicon-cooking" title="Seasoned Advice"></div><a href="https://cooking.stackexchange.com/questions/129893/i-over-salted-my-prime-rib-now-what" class="js-gps-track question-hyperlink mb0" data-gps-track="site.switch({ item_type:11, target_site:49 }); posts_hot_network.click({ item_type:2, location:11 })"> I over salted my prime rib! Now what? </a> </li> <li > <div class="favicon favicon-philosophy" title="Philosophy Stack Exchange"></div><a href="https://philosophy.stackexchange.com/questions/121055/how-does-philosophy-of-religion-deal-with-the-fact-that-there-are-so-many-incomp" class="js-gps-track question-hyperlink mb0" data-gps-track="site.switch({ item_type:11, target_site:265 }); posts_hot_network.click({ item_type:2, location:11 })"> How does philosophy of religion deal with the fact that there are so many incompatible views out there? </a> </li> <li > <div class="favicon favicon-mathematica" title="Mathematica Stack Exchange"></div><a href="https://mathematica.stackexchange.com/questions/309848/how-can-we-keep-each-pair-of-contours-and-removing-others" class="js-gps-track question-hyperlink mb0" data-gps-track="site.switch({ item_type:11, target_site:387 }); posts_hot_network.click({ item_type:2, location:11 })"> How can we keep each pair of contours and removing others? </a> </li> <li > <div class="favicon favicon-superuser" title="Super User"></div><a href="https://superuser.com/questions/1868804/ubuntu-22-04-wifi-adapter-not-listing-access-points" class="js-gps-track question-hyperlink mb0" data-gps-track="site.switch({ item_type:11, target_site:3 }); posts_hot_network.click({ item_type:2, location:11 })"> Ubuntu 22.04 Wifi adapter not listing access points </a> </li> <li > <div class="favicon favicon-travel" title="Travel Stack Exchange"></div><a href="https://travel.stackexchange.com/questions/192836/denial-of-boarding-or-ticketing-issue-best-path-forward" class="js-gps-track question-hyperlink mb0" data-gps-track="site.switch({ item_type:11, target_site:273 }); posts_hot_network.click({ item_type:2, location:11 })"> Denial of boarding or ticketing issue - best path forward </a> </li> <li class="dno js-hidden"> <div class="favicon favicon-scifi" title="Science Fiction & Fantasy Stack Exchange"></div><a href="https://scifi.stackexchange.com/questions/293928/does-tolkien-ever-show-or-speak-of-orcs-being-literate" class="js-gps-track question-hyperlink mb0" data-gps-track="site.switch({ item_type:11, target_site:186 }); posts_hot_network.click({ item_type:2, location:11 })"> Does Tolkien ever show or speak of orcs being literate? </a> </li> <li class="dno js-hidden"> <div class="favicon favicon-linguistics" title="Linguistics Stack Exchange"></div><a href="https://linguistics.stackexchange.com/questions/49636/pronunciation-of-n-in-envy-the-name-of-my-personal-pronunciation-an-approxim" class="js-gps-track question-hyperlink mb0" data-gps-track="site.switch({ item_type:11, target_site:312 }); posts_hot_network.click({ item_type:2, location:11 })"> Pronunciation of N in "envy"? The name of my personal pronunciation, an approximant? </a> </li> <li class="dno js-hidden"> <div class="favicon favicon-law" title="Law Stack Exchange"></div><a href="https://law.stackexchange.com/questions/106666/what-does-within-ten-days-sundays-excepted-the-veto-period-mean-in-art" class="js-gps-track question-hyperlink mb0" data-gps-track="site.switch({ item_type:11, target_site:617 }); posts_hot_network.click({ item_type:2, location:11 })"> What does "within ten Days (Sundays excepted)" — the veto period — mean in Art. I, § 7, Cl. 2 of the US Constitution? </a> </li> <li class="dno js-hidden"> <div class="favicon favicon-aviation" title="Aviation Stack Exchange"></div><a href="https://aviation.stackexchange.com/questions/107614/on-a-light-aircraft-should-i-turn-off-the-anti-collision-light-beacon-strobe-l" class="js-gps-track question-hyperlink mb0" data-gps-track="site.switch({ item_type:11, target_site:528 }); posts_hot_network.click({ item_type:2, location:11 })"> On a light aircraft, should I turn off the anti-collision light (beacon/strobe light) when I stop the engine? </a> </li> <li class="dno js-hidden"> <div class="favicon favicon-diy" title="Home Improvement Stack Exchange"></div><a href="https://diy.stackexchange.com/questions/312283/ceiling-light-emits-a-dim-glow-even-when-turned-off" class="js-gps-track question-hyperlink mb0" data-gps-track="site.switch({ item_type:11, target_site:73 }); posts_hot_network.click({ item_type:2, location:11 })"> Ceiling light emits a dim glow even when turned off </a> </li> <li class="dno js-hidden"> <div class="favicon favicon-scifi" title="Science Fiction & Fantasy Stack Exchange"></div><a href="https://scifi.stackexchange.com/questions/293942/a-3d-animated-movie-about-a-dinosaur-that-survived-to-this-day-and-talks-a-lot" class="js-gps-track question-hyperlink mb0" data-gps-track="site.switch({ item_type:11, target_site:186 }); posts_hot_network.click({ item_type:2, location:11 })"> A 3D-animated movie about a dinosaur that survived to this day and talks a lot </a> </li> <li class="dno js-hidden"> <div class="favicon favicon-scifi" title="Science Fiction & Fantasy Stack Exchange"></div><a href="https://scifi.stackexchange.com/questions/293930/older-scifi-fantasy-story-where-group-take-part-in-a-competition-1-winner-gets" class="js-gps-track question-hyperlink mb0" data-gps-track="site.switch({ item_type:11, target_site:186 }); posts_hot_network.click({ item_type:2, location:11 })"> Older scifi/fantasy story where group take part in a competition, 1 winner gets a wish, the rest lose their soul </a> </li> <li class="dno js-hidden"> <div class="favicon favicon-physics" title="Physics Stack Exchange"></div><a href="https://physics.stackexchange.com/questions/838512/does-noethers-first-theorem-strictly-require-topological-groups-or-lie-groups" class="js-gps-track question-hyperlink mb0" data-gps-track="site.switch({ item_type:11, target_site:151 }); posts_hot_network.click({ item_type:2, location:11 })"> Does Noether's first theorem strictly require topological groups or Lie groups? </a> </li> <li class="dno js-hidden"> <div class="favicon favicon-politics" title="Politics Stack Exchange"></div><a href="https://politics.stackexchange.com/questions/90177/why-did-the-western-world-shift-right-in-post-covid-elections" class="js-gps-track question-hyperlink mb0" data-gps-track="site.switch({ item_type:11, target_site:475 }); posts_hot_network.click({ item_type:2, location:11 })"> Why did the "Western World" shift right in post Covid elections? </a> </li> <li class="dno js-hidden"> <div class="favicon favicon-scifi" title="Science Fiction & Fantasy Stack Exchange"></div><a href="https://scifi.stackexchange.com/questions/293903/1970s-short-story-with-the-last-garden-on-top-of-a-skyscraper-on-a-world-covere" class="js-gps-track question-hyperlink mb0" data-gps-track="site.switch({ item_type:11, target_site:186 }); posts_hot_network.click({ item_type:2, location:11 })"> 1970's short story with the last garden on top of a skyscraper on a world covered in concrete </a> </li> <li class="dno js-hidden"> <div class="favicon favicon-tex" title="TeX - LaTeX Stack Exchange"></div><a href="https://tex.stackexchange.com/questions/733971/tikz-pgf-can-you-set-arrow-size-based-on-the-height-of-the-node-it-is-attached" class="js-gps-track question-hyperlink mb0" data-gps-track="site.switch({ item_type:11, target_site:85 }); posts_hot_network.click({ item_type:2, location:11 })"> TikZ/PGF: Can you set arrow size based on the height of the node it is attached to? </a> </li> <li class="dno js-hidden"> <div class="favicon favicon-philosophy" title="Philosophy Stack Exchange"></div><a href="https://philosophy.stackexchange.com/questions/121068/does-identity-theory-solve-the-hard-problem-of-consciousness" class="js-gps-track question-hyperlink mb0" data-gps-track="site.switch({ item_type:11, target_site:265 }); posts_hot_network.click({ item_type:2, location:11 })"> Does identity theory “solve” the hard problem of consciousness? </a> </li> <li class="dno js-hidden"> <div class="favicon favicon-travel" title="Travel Stack Exchange"></div><a href="https://travel.stackexchange.com/questions/192839/los-angeles-airport-domestic-to-international-transfer-in-90mins" class="js-gps-track question-hyperlink mb0" data-gps-track="site.switch({ item_type:11, target_site:273 }); posts_hot_network.click({ item_type:2, location:11 })"> Los Angeles Airport Domestic to International Transfer in 90mins </a> </li> <li class="dno js-hidden"> <div class="favicon favicon-mathoverflow" title="MathOverflow"></div><a href="https://mathoverflow.net/questions/485159/five-diagonal-determinant-evaluation-with-floor-function" class="js-gps-track question-hyperlink mb0" data-gps-track="site.switch({ item_type:11, target_site:504 }); posts_hot_network.click({ item_type:2, location:11 })"> Five diagonal determinant evaluation with floor function </a> </li> <li class="dno js-hidden"> <div class="favicon favicon-worldbuilding" title="Worldbuilding Stack Exchange"></div><a href="https://worldbuilding.stackexchange.com/questions/263842/a-superhuman-character-only-damaged-by-a-nuclear-blast-s-fireball-what-other-mo" class="js-gps-track question-hyperlink mb0" data-gps-track="site.switch({ item_type:11, target_site:579 }); posts_hot_network.click({ item_type:2, location:11 })"> A superhuman character only damaged by a nuclear blast’s fireball. What other modern or near future weapon could damage them? </a> </li> <li class="dno js-hidden"> <div class="favicon favicon-mathoverflow" title="MathOverflow"></div><a href="https://mathoverflow.net/questions/485085/how-hard-is-the-classification-of-finitely-presented-or-generated-simple-groups" class="js-gps-track question-hyperlink mb0" data-gps-track="site.switch({ item_type:11, target_site:504 }); posts_hot_network.click({ item_type:2, location:11 })"> How hard is the classification of finitely presented or generated simple groups? </a> </li> <li class="dno js-hidden"> <div class="favicon favicon-codegolf" title="Code Golf Stack Exchange"></div><a href="https://codegolf.stackexchange.com/questions/277465/is-it-in-the-sequence-sum-of-the-first-n-cubes" class="js-gps-track question-hyperlink mb0" data-gps-track="site.switch({ item_type:11, target_site:200 }); posts_hot_network.click({ item_type:2, location:11 })"> Is it in the sequence? (sum of the first n cubes) </a> </li> <li class="dno js-hidden"> <div class="favicon favicon-codegolf" title="Code Golf Stack Exchange"></div><a href="https://codegolf.stackexchange.com/questions/277441/distinct-characters-and-distinct-sizes" class="js-gps-track question-hyperlink mb0" data-gps-track="site.switch({ item_type:11, target_site:200 }); posts_hot_network.click({ item_type:2, location:11 })"> Distinct characters and distinct sizes </a> </li> <li class="dno js-hidden"> <div class="favicon favicon-money" title="Personal Finance & Money Stack Exchange"></div><a href="https://money.stackexchange.com/questions/164933/is-online-job-converting-crypto-to-cash-a-scam" class="js-gps-track question-hyperlink mb0" data-gps-track="site.switch({ item_type:11, target_site:93 }); posts_hot_network.click({ item_type:2, location:11 })"> Is online job converting crypto to cash a scam? </a> </li> </ul> <a href="#" class="show-more js-show-more js-gps-track" data-gps-track="posts_hot_network.click({ item_type:3, location:11 })"> more hot questions </a> </div> <div id="feed-link" class="js-feed-link"> <a href="/feeds/question/8456" title="Feed of this question and its answers"> <svg aria-hidden="true" class="fc-orange-400 svg-icon iconRss" width="18" height="18" viewBox="0 0 18 18"><path d="M3 1a2 2 0 0 0-2 2v12c0 1.1.9 2 2 2h12a2 2 0 0 0 2-2V3a2 2 0 0 0-2-2zm0 1.5c6.9 0 12.5 5.6 12.5 12.5H13C13 9.55 8.45 5 3 5zm0 5c4.09 0 7.5 3.41 7.5 7.5H8c0-2.72-2.28-5-5-5zm0 5c1.36 0 2.5 1.14 2.5 2.5H3z"/></svg> Question feed </a> </div> <aside class="s-modal js-feed-link-modal" tabindex="-1" role="dialog" aria-labelledby="feed-modal-title" aria-describedby="feed-modal-description" aria-hidden="true"> <div class="s-modal--dialog js-modal-dialog wmx4" role="document" data-controller="se-draggable"> <h1 class="s-modal--header fw-bold js-first-tabbable" id="feed-modal-title" data-se-draggable-target="handle" tabindex="0"> Subscribe to RSS </h1> <div class="d-flex gs4 gsy fd-column"> <div class="flex--item"> <label class="d-block s-label c-default" for="feed-url"> Question feed <p class="s-description mt2" id="feed-modal-description">To subscribe to this RSS feed, copy and paste this URL into your RSS reader.</p> </label> </div> <div class="d-flex ps-relative"> <input class="s-input" type="text" name="feed-url" id="feed-url" readonly="readonly" value="https://matheducators.stackexchange.com/feeds/question/8456" /> <svg aria-hidden="true" class="s-input-icon fc-orange-400 svg-icon iconRss" width="18" height="18" viewBox="0 0 18 18"><path d="M3 1a2 2 0 0 0-2 2v12c0 1.1.9 2 2 2h12a2 2 0 0 0 2-2V3a2 2 0 0 0-2-2zm0 1.5c6.9 0 12.5 5.6 12.5 12.5H13C13 9.55 8.45 5 3 5zm0 5c4.09 0 7.5 3.41 7.5 7.5H8c0-2.72-2.28-5-5-5zm0 5c1.36 0 2.5 1.14 2.5 2.5H3z"/></svg> </div> </div> <a class="s-modal--close s-btn s-btn__muted js-modal-close js-last-tabbable" href="#" aria-label="Close"> <svg aria-hidden="true" class="svg-icon iconClearSm" width="14" height="14" viewBox="0 0 14 14"><path d="M12 3.41 10.59 2 7 5.59 3.41 2 2 3.41 5.59 7 2 10.59 3.41 12 7 8.41 10.59 12 12 10.59 8.41 7z"/></svg> </a> </div> </aside> </div> </div> <script>StackExchange.ready(function(){$.get('/posts/8456/ivc/5d91?prg=52b539a6-79e8-4add-a499-9ac156a8f381');});</script> <noscript><div><img src="/posts/8456/ivc/5d91?prg=52b539a6-79e8-4add-a499-9ac156a8f381" class="dno" alt="" width="0" height="0"></div></noscript></div> <script>console.error(`Unable to load nonexistent manifest entry 'recent-posts'. Check that your file path is correct.`)</script> </div> </div> <script type="text/javascript"> var cam = cam || { opt: {} }; var clcGamLoaderOptions = cam || { opt: {} }; var opt = clcGamLoaderOptions.opt; opt.refresh = !1; opt.refreshInterval = 0; opt.sf = !1; opt.hb = !1; opt.ll = !0; opt.tlb_position = 0; opt.personalization_consent = !1; opt.targeting_consent = !1; opt.performance_consent = !1; opt.targeting = {Registered:['false'],Reputation:['new'],NumberOfAnswers:['4'],cf_bot_score:'1'}; opt.adReportEnabled = !1; opt.adReportUrl = '/ads/report-ad'; opt.adReportText = 'Report this ad'; opt.adReportFileTypeErrorMessage = 'Please select a PNG or JPG file.'; opt.adReportFileSizeErrorMessage = 'The file must be under 2 MiB.'; opt.adReportErrorText = 'Error uploading ad report.'; opt.adReportThanksText = 'Thanks for your feedback. We’ll review this against our code of conduct and take action if necessary.'; opt.adReportLoginExpiredMessage = 'Your login session has expired, please login and try again.'; opt.adReportLoginErrorMessage = 'An error occurred when loading the report form - please try again'; opt.adReportModalClass = 'js-ad-report'; opt.countryCode = 'SG'; opt.qualtricsSurveyData = '{"isRegistered":"False","repBucket":"new","referrer":"https%3a%2f%2fmatheducators.stackexchange.com%2fquestions%2f8456%2finquiry-about-my-note-taking-skill%2f8475","accountAge":"0"}'; opt.perRequestGuid = '52b539a6-79e8-4add-a499-9ac156a8f381'; opt.responseHash = 'rQ5CpHLkt0SUo3Ig2aakgyweoWxz26EzKRwMlRt2J9o='; opt.targeting.TargetingConsent = ['False_Passive']; opt.allowAccountTargetingForThisRequest = !1; const urlParams = new URLSearchParams(window.location.search); if (urlParams.has('dfptestads')) { const dfptestads = urlParams.get('dfptestads'); opt.targeting.DfpTestAds = dfptestads; } </script> <script>;(()=>{"use strict";var __webpack_modules__={23:(e,t,s)=>{s.d(t,{Z7:()=>c,eq:()=>l,kG:()=>d});const n="248424177",o=(a=location.pathname,/^\/tags\//.test(a)||/^\/questions\/tagged\//.test(a)?"tag-pages":/^\/discussions\//.test(a)||/^\/beta\/discussions/.test(a)?"discussions":/^\/$/.test(a)||/^\/home/.test(a)?"home-page":/^\/jobs$/.test(a)||/^\/jobs\//.test(a)?"jobs":"question-pages");var a;let i=location.hostname;const r={slots:{lb:[[728,90]],mlb:[[728,90]],smlb:[[728,90]],bmlb:[[728,90]],sb:e=>"dfp-tsb"===e?[[300,250],[300,600]]:[[300,250]],"tag-sponsorship":[[730,135]],"mobile-below-question":[[320,50],[300,250]],msb:[[300,250],[300,600]],"talent-conversion-tracking":[[1,1]],"site-sponsorship":[[230,60]]},ids:{"dfp-tlb":"lb","dfp-mlb":"mlb","dfp-smlb":"smlb","dfp-bmlb":"bmlb","dfp-tsb":"sb","dfp-isb":"sb","dfp-tag":"tag-sponsorship","dfp-msb":"msb","dfp-sspon":"site-sponsorship","dfp-m-aq":"mobile-below-question"},idsToExcludeFromAdReports:["dfp-sspon"]};function d(){return Object.keys(r.ids)}function l(e){return r.idsToExcludeFromAdReports.indexOf(e)<0}function c(e){var t=e.split("_")[0];const s=r.ids[t];let a=r.slots[s];return"function"==typeof a&&(a=a(t)),{path:`/${n}/${i}/${s}/${o}`,sizes:a,zone:s}}},865:(e,t,s)=>{function n(e){return"string"==typeof e?document.getElementById(e):e}function o(e){return!!(e=n(e))&&"none"===getComputedStyle(e).display}function a(e){return!o(e)}function i(e){return!!e}function r(e){return/^\s*$/.test(n(e).innerHTML)}function d(e){const{style:t}=e;t.height=t.maxHeight=t.minHeight="auto",t.display="none"}function l(e){const{style:t}=e;t.height=t.maxHeight=t.minHeight="auto",t.display="none",[].forEach.call(e.children,l)}function c(e){const{style:t}=e;t.height=t.maxHeight=t.minHeight="auto",t.removeProperty("display")}function g(e){const t=document.createElement("script");t.src=e,document.body.appendChild(t)}function p(e){return s=e,(t=[]).push=function(e){return s(),delete this.push,this.push(e)},t;var t,s}function h(e){let t="function"==typeof HTMLTemplateElement;var s=document.createElement(t?"template":"div");return e=e.trim(),s.innerHTML=e,t?s.content.firstChild:s.firstChild}s.d(t,{$Z:()=>c,Bv:()=>h,Gx:()=>g,Nj:()=>n,QZ:()=>p,cf:()=>d,pn:()=>a,wo:()=>l,xb:()=>r,xj:()=>o,yb:()=>i})},763:(__unused_webpack_module,__webpack_exports__,__webpack_require__)=>{__webpack_require__.d(__webpack_exports__,{t:()=>AdReports});var _common_helper__WEBPACK_IMPORTED_MODULE_2__=__webpack_require__(865),_console__WEBPACK_IMPORTED_MODULE_1__=__webpack_require__(276),_ad_units__WEBPACK_IMPORTED_MODULE_0__=__webpack_require__(23);class AdReports{constructor(e,t){if(this.googletag=e,this.cam=t,this.allowedFileTypes=["image/png","image/jpg","image/jpeg"],this.ignoreValidation=!1,_console__WEBPACK_IMPORTED_MODULE_1__.cM("Ad reporting init"),this.cam=t,this.callOnButtonClick=e=>this.onButtonClick(e),this.googletag.pubads().addEventListener("slotRenderEnded",e=>this.handleSlotRendered(e)),Array.isArray(t.slotsRenderedEvents)){_console__WEBPACK_IMPORTED_MODULE_1__.cM("Adding report button to "+t.slotsRenderedEvents.length+" events that have transpired");for(var s=0;s<t.slotsRenderedEvents.length;s++)this.handleSlotRendered(t.slotsRenderedEvents[s])}}handleSlotRendered(e){if(e&&e.slot&&!e.isEmpty&&(e.creativeId||e.lineItemId||!e.isEmpty)){var t=e.slot.getSlotElementId();if(t){var s=document.getElementById(t);if(s)if((0,_ad_units__WEBPACK_IMPORTED_MODULE_0__.eq)(t)){var n=s?.closest(".js-zone-container")?.querySelector(".js-report-ad-button-container");n?(n.innerHTML="",n.append(this.createButton(e)),n.style.height="24px",_console__WEBPACK_IMPORTED_MODULE_1__.cM("Added report button to the bottom of "+t)):_console__WEBPACK_IMPORTED_MODULE_1__.cM("Ad report button not found, may be intentional, element: "+t)}else _console__WEBPACK_IMPORTED_MODULE_1__.cM("Not adding report button to the bottom of "+t+": shouldHaveReportButton = false");else _console__WEBPACK_IMPORTED_MODULE_1__.cM("Not adding report button to the bottom of "+t+": resolved invalid adUnit element")}else _console__WEBPACK_IMPORTED_MODULE_1__.cM("Not adding report button to the bottom of element: invalid adUnitElementId")}else _console__WEBPACK_IMPORTED_MODULE_1__.cM("Not adding report button to the bottom of element: invalid SlotRenderEndedEvent")}async onButtonClick(e){e.preventDefault();let t=e.target;const s=t.dataset.modalUrl,n=t.dataset.googleEventData;return await this.loadModal(s,t,n),!1}createButton(e){let t=document.createElement("button");var s=JSON.stringify(e);return t.dataset.googleEventData=s,t.dataset.modalUrl=this.cam.opt.adReportUrl,t.dataset.adUnit=e.slot.getSlotElementId(),t.classList.add("js-report-ad","s-btn","s-btn__link","fs-fine","mt2","float-right"),t.append(document.createTextNode(this.cam.opt.adReportText)),t.removeEventListener("click",this.callOnButtonClick),t.addEventListener("click",this.callOnButtonClick),t}async loadModal(url,$link,googleEventData){try{await window.StackExchange.helpers.loadModal(url,{returnElements:window.$($link)}),this.initForm(googleEventData)}catch(e){var message="",response=e.responseText?eval(`(${e.responseText})`):null;message=response&&response.isLoggedOut?this.cam.opt.adReportLoginExpiredMessage:this.cam.opt.adReportLoginErrorMessage,window.StackExchange.helpers.showToast(message,{type:"danger"})}}removeModal(){window.StackExchange.helpers.closePopups(document.querySelectorAll("."+this.cam.opt.adReportModalClass),"dismiss")}initForm(e,t=!1){this.ignoreValidation=t,this.$form=document.querySelector(".js-ad-report-form"),this.$googleEventData=this.$form.querySelector(".js-json-data"),this.$adReportReasons=this.$form.querySelectorAll(".js-ad-report-reason"),this.$adReportReasonOther=this.$form.querySelector(".js-ad-report-reason-other"),this.$fileUploaderInput=this.$form.querySelector(".js-file-uploader-input"),this.$imageUploader=this.$form.querySelector(".js-image-uploader"),this.$clearImageUpload=this.$form.querySelector(".js-clear-image-upload"),this.$imageUploaderText=this.$form.querySelector(".js-image-uploader-text"),this.$imageUploaderPreview=this.$form.querySelector(".js-image-uploader-preview"),this.$fileErrorMessage=this.$form.querySelector(".js-file-error");const s=this.$form.querySelector(".js-drag-drop-enabled"),n=this.$form.querySelector(".js-drag-drop-disabled");this.$googleEventData.value=e,this.$adReportReasons.forEach((e,t)=>e.addEventListener("change",e=>{this.$adReportReasonOther.classList.toggle("d-none","3"!==e.target.value)})),this.$fileUploaderInput.addEventListener("change",()=>{this.validateFileInput()&&this.updateImagePreview(this.$fileUploaderInput.files)}),this.$clearImageUpload.addEventListener("click",e=>{e.preventDefault(),this.clearImageUpload()});try{this.$fileUploaderInput[0].value="",this.$imageUploader.addEventListener("dragenter dragover dragleave drop",this.preventDefaults),this.$imageUploader.addEventListener("dragenter dragover",this.handleDragStart),this.$imageUploader.addEventListener("dragleave drop",this.handleDragEnd),this.$imageUploader.addEventListener("drop",this.handleDrop)}catch(e){s.classList.add("d-none"),n.classList.remove("d-none")}this.$form.removeEventListener("",this.handleDragEnd),this.$form.addEventListener("submit",async e=>(e.preventDefault(),this.submitForm(),!1))}clearImageUpload(){this.$fileUploaderInput.value="",this.$imageUploaderPreview.setAttribute("src",""),this.$imageUploaderPreview.classList.add("d-none"),this.$clearImageUpload.classList.add("d-none"),this.$imageUploaderText.classList.remove("d-none"),this.$imageUploader.classList.add("p16","ba","bas-dashed","bc-black-100")}preventDefaults(e){e.preventDefault(),e.stopPropagation()}handleDragStart(e){this.$imageUploader.classList.remove("bas-dashed"),this.$imageUploader.classList.add("bas-solid","bc-black-100")}handleDragEnd(e){this.$imageUploader.classList.remove("bas-solid","bc-black-100"),this.$imageUploader.classList.add("bas-dashed")}handleDrop(e){var t=e.originalEvent.dataTransfer.files;FileReader&&t&&1===t.length&&(this.$fileUploaderInput.files=t,this.validateFileInput()&&this.updateImagePreview(t))}setError(e){this.$fileErrorMessage.parentElement.classList.toggle("has-error",e)}updateImagePreview(e){this.$imageUploader.classList.remove("p16","ba","bas-dashed","bc-black-100"),this.$clearImageUpload.classList.remove("d-none"),this.$imageUploaderText.classList.add("d-none");var t=new FileReader;t.onload=e=>{null!=e.target&&(this.$imageUploaderPreview.setAttribute("src",e.target.result),this.$imageUploaderPreview.classList.remove("d-none"))},t.readAsDataURL(e[0])}validateFileInput(){if(this.ignoreValidation)return!0;const e=this.cam.opt.adReportFileTypeErrorMessage,t=this.cam.opt.adReportFileSizeErrorMessage;if(null==this.$fileUploaderInput.files)return!1;var s=this.$fileUploaderInput.files[0];return null==s?(this.setError(!0),!1):this.allowedFileTypes.indexOf(s.type)<0?(this.$fileErrorMessage.textContent=e,this.$fileErrorMessage.classList.remove("d-none"),this.setError(!0),!1):s.size>2097152?(this.$fileErrorMessage.textContent=t,this.$fileErrorMessage.classList.remove("d-none"),this.setError(!0),!1):(this.$fileErrorMessage.classList.add("d-none"),this.setError(!1),!0)}async gatherDiagnosticInfo(){return{BrowserVersion:await this.getBrowserVersion()}}getElementSource(e){return e.outerHTML}getNestedIFrameElement(e){var t=e.querySelector("iframe");return t.contentDocument?t.contentDocument.documentElement:t.contentWindow.document.documentElement}async getBrowserVersion(){return await navigator.userAgentData.getHighEntropyValues(["fullVersionList"]).then(e=>JSON.stringify(e.fullVersionList))}async submitForm(){if(!this.validateFileInput())return!1;this.$form.querySelector("[type=submit]").setAttribute("disabled","true");var e=JSON.parse(this.$googleEventData.value||"{}");e.Reason=parseInt(this.$form.querySelector(".js-ad-report-reason:checked").value,10),e.Description=this.$adReportReasonOther.value,this.$googleEventData.value=JSON.stringify(e);var t=new FormData(this.$form);if("1"===t.get("shareDiagnosticInfo")){var s=await this.gatherDiagnosticInfo();Object.keys(s).forEach(e=>t.append(e,s[e]))}try{const e=await window.fetch(this.$form.getAttribute("action"),{method:this.$form.getAttribute("method"),body:t,cache:"no-cache"}),s=e.headers.get("content-type")||"",o=await e.text();if(!e.ok)throw new Error("response not valid");if(0===s.indexOf("text/html")){var n=(0,_common_helper__WEBPACK_IMPORTED_MODULE_2__.Bv)(o);const e=n?n.querySelector(".js-modal-content"):null;if(_console__WEBPACK_IMPORTED_MODULE_1__.cM("$popupContent"),_console__WEBPACK_IMPORTED_MODULE_1__.cM(e),!e)throw new Error(`Could not find .js-modal-content in response from ${this.$form.getAttribute("action")}`);document.querySelector(".js-modal-content").replaceWith(e)}else window.StackExchange.helpers.showToast(this.cam.opt.adReportThanksText,{type:"success"}),this.removeModal()}catch(e){window.StackExchange.helpers.showToast(this.cam.opt.adReportErrorText,{type:"danger"})}finally{let e=this.$form.querySelector("[type=submit]");e&&e.removeAttribute("disabled")}}}},276:(e,t,s)=>{function n(...e){}function o(...e){}s.d(t,{cM:()=>n,vU:()=>o})}},__webpack_module_cache__={};function __webpack_require__(e){var t=__webpack_module_cache__[e];if(void 0!==t)return t.exports;var s=__webpack_module_cache__[e]={exports:{}};return __webpack_modules__[e](s,s.exports,__webpack_require__),s.exports}__webpack_require__.d=(e,t)=>{for(var s in t)__webpack_require__.o(t,s)&&!__webpack_require__.o(e,s)&&Object.defineProperty(e,s,{enumerable:!0,get:t[s]})},__webpack_require__.o=(e,t)=>Object.prototype.hasOwnProperty.call(e,t);var __webpack_exports__={};(()=>{var e=__webpack_require__(276),t=(e=>(e[e.Above=0]="Above",e[e.Below=1]="Below",e))(t||{});const s=Object.assign({},{"lib":"https://clc.stackoverflow.com/Content/bundles/js/gam_loader_script.bundle.741.25e3e63be9a306287d9f.js?v=d5f908ccaade","style":null,"u":null,"wa":true,"kt":2000,"tto":true,"h":"clc.stackoverflow.com","allowed":"^(((talent\\.)?stackoverflow)|(blog\\.codinghorror)|(.*\\.googlesyndication)|(serverfault|askubuntu|superuser)|([^\\.]+\\.stackexchange))\\.com$","wv":true,"al":false,"abd":true,"cpa_liid":[5882654614],"cpa_cid":[138377597667],"dp":false,"tgt_to":1000,"tgt_u":"https://clc.stackoverflow.com/get-user-acct-tgt","tgt_e":true,"tgt_p":100,"dv_enabled":false});var n=__webpack_require__(23),o=__webpack_require__(865),a=__webpack_require__(763);class i{constructor(t,s){this.googletag=t,this.interval=s,e.cM("Ad refresh init. interval: "+s),this.googletag.pubads().addEventListener("impressionViewable",e=>this.onImpressionViewable(e)),e.cM("done enabling ad refresh")}onImpressionViewable(t){var s=t.slot;e.cM("ad refresh - slot "+s.getSlotElementId()+" is viewable, initializing refresh"),this.scheduleRefresh(s)}scheduleRefresh(e){setTimeout(()=>this.refreshAdSlot(e),1e3*this.interval)}static refreshMyAd(t,s){let n=t.pubads().getSlots().find(e=>e.getSlotElementId()===s);n&&(e.cM("refreshMyAd - refreshing ad slot "+s),t.pubads().refresh([n]))}static removeMyAd(t,s){let n=t.pubads().getSlots().find(e=>e.getSlotElementId()===s);n&&(e.cM("removeMyAd - destroying ad slot "+s),t.destroySlots([n]))}refreshAdSlot(t){var s=t.getSlotElementId();this.isElementVisibleInBrowser(s)?(e.cM("refreshing ad slot "+s),googletag.pubads().refresh([t])):(e.cM("refresh skipped this time; ad slot not viewable:"+s),this.scheduleRefresh(t))}isElementVisibleInBrowser(e){var t=document.getElementById(e);if(null!==t){var s=t.getBoundingClientRect();if(s.top>=0&&s.left>=0&&s.bottom<=(window.innerHeight||document.documentElement.clientHeight)&&s.right<=(window.innerWidth||document.documentElement.clientWidth))return!0}return!1}}var r=(e=>(e.Off="Off",e.PreSurvey="PreSurvey",e.Collect="Collect",e.PostSurvey="PostSurvey",e))(r||{});class d{constructor(e,t){this.lineItemImpressions=[],this.surveysIdsCompleted=[],this.lineItemImpressions=e,this.surveysIdsCompleted=t}addImpression(e,t){let s={brandId:e,lineItemId:t,timestamp:new Date};this.lineItemImpressions.push(s)}addBrandSurveyCompleted(e){-1===this.surveysIdsCompleted.indexOf(e)&&this.surveysIdsCompleted.push(e)}getTotalBrandImpressions(){let e=new Map;for(let t of this.lineItemImpressions)if(e.has(t.brandId)){let s=e.get(t.brandId);e.set(t.brandId,s+1)}else e.set(t.brandId,1);return e}getBrandLineItemImpressions(e){let t={};for(let s of this.lineItemImpressions)if(s.brandId==e)if(void 0!==t[s.lineItemId]){let e=t[s.lineItemId];t[s.lineItemId]=e+1}else t[s.lineItemId]=1;return t}}class l{constructor(){this.surveyEngagementLocalStorageKey="clc-survey-engagement"}getBrandSurveyEngagement(){let e=localStorage.getItem(this.surveyEngagementLocalStorageKey);if(null===e)return new d([],[]);let t=JSON.parse(e);return new d(t.lineItemImpressions,t.surveysIdsCompleted)}saveBrandSurveyEngagement(e){let t=JSON.stringify(e);localStorage.setItem(this.surveyEngagementLocalStorageKey,t)}}class c{constructor(){this.surveyRepository=new l}getBrandSurveyEngagement(){return this.surveyRepository.getBrandSurveyEngagement()}recordImpression(e,t){let s=this.getBrandSurveyEngagement();s.addImpression(e,t),this.surveyRepository.saveBrandSurveyEngagement(s)}recordBrandSurveyCompleted(e){let t=this.getBrandSurveyEngagement();t.addBrandSurveyCompleted(e),this.surveyRepository.saveBrandSurveyEngagement(t)}}class g{constructor(t,s){this.googletag=t,this.brandSettings=s,this.brandSlotMap=new Map,this.brandSurveyEngagementService=new c,e.cM("Brand Survey init: "+JSON.stringify(s)),void 0!==s?(this.googletag.pubads().addEventListener("slotRenderEnded",e=>this.handleSlotRendered(e)),this.googletag.pubads().addEventListener("impressionViewable",e=>this.onImpressionViewable(e)),e.cM("done enabling Brand Survey")):e.cM("Brand Survey init: brandSettings is undefined, not initializing")}handleSlotRendered(t){e.cM("Brand Survey - slot rendered - slot:"+JSON.stringify(t.slot.getSlotElementId())+" lineItem: "+t.lineItemId);let s=this.findItemWithId(t.lineItemId);if(null===s||s.mode!==r.Collect)this.brandSlotMap.delete(t.slot.getSlotElementId());else{let e={brandId:s.brandId,lineItemId:t.lineItemId};this.brandSlotMap.set(t.slot.getSlotElementId(),e)}}onImpressionViewable(t){let s=t.slot;if(e.cM("ad - Brand Survey - impression viewable. Details: "+JSON.stringify(s.getSlotElementId())),e.cM("ad - Brand Survey - slot "+s.getSlotElementId()+" is viewable"),this.brandSlotMap.has(s.getSlotElementId())){let t=this.brandSlotMap.get(s.getSlotElementId());e.cM("Brand Survey - brand "+t.brandId+" is viewable"),this.recordImpression(this.brandSlotMap.get(s.getSlotElementId()))}}recordImpression(t){e.cM("ad - Brand Survey - recording impression for brand "+t.brandId),this.brandSurveyEngagementService.recordImpression(t.brandId,t.lineItemId)}findItemWithId(t){return e.cM("brand settings: "+JSON.stringify(this.brandSettings)),this.brandSettings.find(e=>e.lineItemIds.includes(t))||null}}const p="response-brand-survey-submit|",h="request-brand-survey-metadata|",m="record-metric-on-server|",u="request-dsp-tags",f="response-dsp-tags|";class v{static refreshAdIfBrandSurveyIsDuplicated(e,t,s){if(this.alreadyCompletedThisBrandSurvey(t)){var n=document.getElementById(s).closest(".js-zone-container");i.removeMyAd(e,s),n&&n.remove()}}static alreadyCompletedThisBrandSurvey(e){return(new c).getBrandSurveyEngagement().surveysIdsCompleted.includes(e)}}window.cam=new class{constructor(t=null){if(this.gptImported=!1,this.slotsRenderedEvents=[],this.collapsed={},e.cM("constructor"),this.clc_options=s,window.clcGamLoaderOptions)Object.assign(this,window.clcGamLoaderOptions);else if(void 0===this.opt){let e=window.opt;e&&(this.opt=e)}}init(){if(e.cM("init"),void 0===this.opt)throw new Error("opt not set, required by GAM Loader");e.cM("init brand survey service"),this.getUserMetaPromise=this.getUserMeta(),e.cM("setup message handler"),window.addEventListener("message",e=>{this.onmessage(e)})}handleSlotRenderedNoAdReport(){if(googletag.pubads().addEventListener("slotRenderEnded",e=>this.applyExtraMarginBottom(e)),Array.isArray(this.slotsRenderedEvents))for(var e=0;e<this.slotsRenderedEvents.length;e++)this.applyExtraMarginBottom(this.slotsRenderedEvents[e])}onmessage(t){let s="omni";if(t.data&&("string"==typeof t.data||t.data instanceof String))if(0===t.data.indexOf("get-omni-")){e.cM("Recevied get-omni message, sending back omni");var n=t.source,a=this.opt.omni,i="string"==typeof a?a:"";n.postMessage([s,i,this.opt.perRequestGuid].join("|"),"*")}else if(0===t.data.indexOf("collapse-")){e.cM("Recevied collapse message, collapse ad iframe"),e.cM(t);for(var r=t.source.window,d=document.getElementsByTagName("IFRAME"),l=0;l<d.length;l++){var g=d[l];if(g.contentWindow==r)return void(0,o.wo)(g.parentElement.parentElement.parentElement)}}else if(0===t.data.indexOf("resize|")){e.cM("Recevied resize message, resize ad iframe"),e.cM(t);let s=this._getFrameByEvent(t),n=t.data.indexOf("|")+1,o=t.data.slice(n),a=parseFloat(o)+.5;e.cM("New iframe height "+a),s.height=a.toString(),s.parentElement.style.height=a.toString()+"px"}else if(0===t.data.indexOf("getmarkup|")){let s=t.data.indexOf("|")+1,n=t.data.slice(s);e.cM("Recevied get markup message: "+n);let o=this._getFrameByEvent(t).closest(".everyonelovesstackoverflow");const a=document.createElement("script");a.dataset.adZoneId=o.id,a.src=n,document.body.appendChild(a)}else if(0===t.data.indexOf("window-location|")){let s=t.data.indexOf("|")+1,n=t.data.slice(s);e.cM("Recevied window location message: "+n),n.startsWith("/")||(n="/"+n),window.open(window.location.protocol+"//"+window.location.host+n,"_blank")}else if(0===t.data.indexOf("request-brand-survey-submit|")){let s=t.data.split("|"),n=s[1],o=s[2],a=s[3],i=JSON.parse(a);e.cM(n),e.cM(o),e.cM(a),e.cM("Received brand survey "+n+" response message: "+o);var _=new FormData;for(var b in i)_.append(b,i[b]);let r=this._getFrameByEvent(t);if(v.alreadyCompletedThisBrandSurvey(+n))return e.cM("Already completed this brand survey. Not submitting duplicate to server."),void r.contentWindow.postMessage("response-brand-survey-submit-duplicate|","*");e.cM("Send the brand survey to the server"),fetch(o,{method:"POST",body:_}).then(e=>e.json()).then(e=>r.contentWindow.postMessage({messageType:p},"*")).catch(e=>r.contentWindow.postMessage({messageType:p},"*"))}else if(0===t.data.indexOf("brand-survey-completed-store|")){let s=t.data.split("|"),n=(s[1],s[2]);if(e.cM("Received brand survey completed store message for survey ID "+n),v.alreadyCompletedThisBrandSurvey(+n))return void e.cM("Already completed this brand survey. Not recording duplicate locally.");e.cM("Record brand survey completion locally"),(new c).recordBrandSurveyCompleted(+n)}else if(0===t.data.indexOf(h)){let s=t.data.split("|"),n=s[1],o=s[2];e.cM("Received message: "+h+" with Brand Survey ID "+o);let a=(new c).getBrandSurveyEngagement().getBrandLineItemImpressions(+n),i=JSON.stringify(a),r=this._getFrameByEvent(t);e.cM("sending impression data: "+i),r.contentWindow.postMessage("response-brand-survey-metadata|"+this.opt.responseHash+"|"+this.opt.perRequestGuid+"|"+i+"|"+this.opt.countryCode+"|"+this.opt.qualtricsSurveyData,"*")}else if(0===t.data.indexOf("refresh-if-duplicate-brand-survey|")){let e=t.data.split("|")[1],s=this.getSlotElementIdByEvent(t);v.refreshAdIfBrandSurveyIsDuplicated(googletag,+e,s)}else if(0===t.data.indexOf(m)){e.cM("Received message: "+m+" with args: "+t.data);let s=t.data.split("|"),n=s[1],o=s[2],a=s[3],i=s[4],r=new FormData;r.append("brandSurveyId",a.toString()),r.append("responseHash",this.opt.responseHash),r.append("perRequestGuid",this.opt.perRequestGuid),r.append("questionNumber",n.toString()),r.append("metricType",i.toString()),fetch(o,{method:"POST",body:r}).then(e=>e.ok).catch(t=>{e.cM("SendMetricToServer: Error sending metric to server: "+t)})}else if(0===t.data.indexOf(u)){e.cM("Received message: "+u+" with args: "+t.data);let s=this._getFrameByEvent(t);if(!this.opt.targeting["so-tag"])return void s.contentWindow.postMessage(f,"*");const n=this.opt.targeting["so-tag"].join(",");e.cM("sending targeting tags: "+n),s.contentWindow.postMessage(f+n,"*")}else e.cM("Received unhandled message")}getSlotElementIdByEvent(e){let t=this._getFrameByEvent(e),s=t.parentElement?.parentElement?.id;return s||""}_getFrameByEvent(e){return Array.from(document.getElementsByTagName("iframe")).filter(t=>t.contentWindow===e.source)[0]}classifyZoneIds(e){const t=e.map(o.Nj).filter(o.yb);return{eligible:t.filter(o.xb).filter(o.pn),ineligible:t.filter(o.xj)}}applyExtraMarginBottom(t){if(t&&t.slot&&!t.isEmpty&&(t.creativeId||t.lineItemId||!t.isEmpty)){var s=t.slot.getSlotElementId();if(s){var o=document.getElementById(s);if(o)if((0,n.eq)(s)){var a=o?.closest(".js-zone-container");a.style.marginBottom="24px",e.cM("Applied extra margin to the bottom of "+s)}else e.cM("Not applying extra margin to the bottom of "+s+": shouldHaveReportButton = false");else e.cM("Not applying extra margin to the bottom of "+s+": resolved invalid adUnit element")}else e.cM("Not applying extra margin to the bottom of element: invalid adUnitElementId")}else e.cM("Not applying extra margin to the bottom of element: invalid SlotRenderEndedEvent")}async load(s=(0,n.kG)()){const r=this.opt.tlb_position===t.Above?["dfp-mlb","dfp-smlb"]:["dfp-mlb","dfp-smlb","dfp-tlb"];if(!this.isGptReady())return e.cM("Initializing..."),this.initGpt(),void googletag.cmd.push(()=>this.load(s));this.opt.adReportEnabled?(e.cM("Ad reporting enabled"),this.adReports=new a.t(googletag,this)):(e.cM("Ad reporting not enabled"),this.handleSlotRenderedNoAdReport()),this.opt.refresh?(e.cM("Ad refresh enabled"),this.adRefresh=new i(googletag,this.opt.refreshInterval)):e.cM("Ad refresh not enabled"),this.opt.brandSurveyEnabled&&(e.cM("Brand Survey enabled"),this.brandSurvey=new g(googletag,this.opt.brandSurveySettings)),e.cM("Attempting to load ads into ids: ",s);const{eligible:d,ineligible:l}=this.classifyZoneIds(s);if(this.initDebugPanel(googletag,d.concat(l)),d.forEach(e=>(0,o.cf)(e)),l.forEach(o.wo),0===d.length)return void e.cM("Found no ad ids on page");e.cM("Eligible ids:",d),this.opt.abd&&this.appendAdblockDetector();var c=googletag.pubads().getSlots();if(c){var p=c.filter(e=>s.indexOf(e.getSlotElementId())>=0);googletag.destroySlots(p)}this.opt.sf&&(googletag.pubads().setForceSafeFrame(!0),googletag.pubads().setSafeFrameConfig({allowOverlayExpansion:!0,allowPushExpansion:!0,sandbox:!0})),e.cM("Targeting consent: Checking...");let h=!1,m=!1;void 0!==this.opt.targeting_consent&&(m=!0,e.cM("Targeting consent: Parameter set"),e.cM("Targeting consent: Consent given? ",this.opt.targeting_consent),h=this.opt.targeting_consent),void 0!==this.opt.personalization_consent&&(e.cM("Personalization consent: Parameter set"),e.cM("Personalization consent: Consent given? ",this.opt.personalization_consent),h=h&&this.opt.personalization_consent),h=h&&m,this.setPrivacySettings(h),this.opt.ll||googletag.pubads().enableSingleRequest(),cam.sreEvent||(googletag.pubads().addEventListener("slotRenderEnded",e=>this.onSlotRendered(e)),cam.sreEvent=!0),await this.setTargeting();var u=d.filter(e=>!this.opt.ll||r.indexOf(e.id)<0),f=d.filter(e=>!!this.opt.ll&&r.indexOf(e.id)>=0);e.cM("Up front ids:",u),e.cM("Lazy loaded ids:",f),u.forEach(t=>{e.cM(`Defining ad for element ${t.id}`),this.defineSlot(t.id,googletag),t.setAttribute("data-dfp-zone","true")}),googletag.enableServices(),u.forEach(t=>{e.cM(`Displaying ad for element ${t.id}`),this.clc_options.dv_enabled?window.onDvtagReady(function(){googletag.display(t.id)}):googletag.cmd.push(()=>googletag.display(t.id))}),this.opt.ll&&(e.cM("Enabling lazy loading for GAM"),googletag.pubads().enableLazyLoad({fetchMarginPercent:0,renderMarginPercent:0}),e.cM("Setting up lazy loaded ad units"),f.forEach(t=>{e.cM(`Lazy loading - Defining Slot ${t.id}`),this.defineSlot(t.id,googletag)}),f.forEach(t=>{e.cM(`Lazy loading - Displaying ad for element ${t.id}`),this.clc_options.dv_enabled?window.onDvtagReady(function(){googletag.display(t.id)}):googletag.cmd.push(()=>googletag.display(t.id))}))}setPrivacySettings(e){e||googletag.pubads().setPrivacySettings({nonPersonalizedAds:!0})}async setTargeting(){if(!googletag)throw new Error("googletag not defined");let t=this.opt.targeting;if(!t)throw new Error("Targeting not defined (is "+typeof t+")");Object.keys(t).forEach(s=>{e.cM(`-> targeting - ${s}: ${t[s]}`),googletag.pubads().setTargeting(s,t[s])});let s=!1;if(void 0!==this.opt.targeting_consent&&(s=this.opt.targeting_consent),s){let t=(new c).getBrandSurveyEngagement();if(t.getTotalBrandImpressions().forEach((t,s)=>{e.cM(`-> targeting - BrandImpressions: ${s}: ${t}`),googletag.pubads().setTargeting("brand_"+s.toString()+"_impressions",t.toString())}),t.surveysIdsCompleted.forEach(t=>{e.cM(`-> targeting - SurveysTaken: ${t}`),googletag.pubads().setTargeting("survey_"+t+"_taken","true")}),this.clc_options.tgt_e&&this.getUserMetaPromise){let t=await this.getUserMetaPromise;t&&t.tgt_acct?(e.cM("-> targeting - User Account: "+t.tgt_acct),googletag.pubads().setTargeting("user-acct",t.tgt_acct.company_name),googletag.pubads().setTargeting("user_acct_top",t.tgt_acct.company_name),googletag.pubads().setTargeting("user_industry",t.tgt_acct.industry),googletag.pubads().setTargeting("user_employee_count",t.tgt_acct.employee_range)):e.cM("-> targeting - User Account: Not Found"),t&&Object.prototype.hasOwnProperty.call(t,"is_high_rep_earner")?(e.cM("-> targeting - High Rep Earner: "+t.is_high_rep_earner),googletag.pubads().setTargeting("IsHighRepEarner",t.is_high_rep_earner?"true":"false")):e.cM("-> targeting - High Rep Earner: not found")}if(localStorage){e.cM('Checking local storage for "jobs-last-clicked" key.');let t=localStorage.getItem("jobs-last-clicked")?"true":"false";e.cM(`-> targeting - jobs_clicked: ${t}`),googletag.pubads().setTargeting("jobs_clicked",t)}}}appendAdblockDetector(){const e=document.createElement("div");e.className="adsbox",e.id="clc-abd",e.style.position="absolute",e.style.pointerEvents="none",e.innerHTML=" ",document.body.appendChild(e)}onSlotRendered(s){try{const i=s.slot.getSlotElementId();let r=[];i||r.push("id=0");const d=document.getElementById(i);if(i&&!d&&r.push("el=0"),0!==r.length)return void this.stalled(r.join("&"));const{path:l,sizes:c,zone:g}=(0,n.Z7)(i);if(this.collapsed[g]&&s.isEmpty)return e.cM(`No line item for the element #${d.id}... collapsing.`),void(0,o.wo)(d);if(this.slotsRenderedEvents.push(s),s.lineItemId||s.creativeId||!s.isEmpty){e.cM(`Rendered ad for element #${d.id} [line item #${s.lineItemId}]`),e.cM(s);var a=d.parentElement;if(a.classList.contains("js-zone-container")){switch((0,o.cf)(a),i){case"dfp-tlb":this.opt.tlb_position===t.Above?a.classList.add("mb8"):a.classList.add("mt16");break;case"dfp-tag":a.classList.add("mb8");break;case"dfp-msb":a.classList.add("mt16");break;case"dfp-mlb":case"dfp-smlb":case"dfp-bmlb":a.classList.add("my8");break;case"dfp-isb":a.classList.add("mt24");break;case"dfp-m-aq":a.classList.add("my12"),a.classList.add("mx-auto")}(0,o.$Z)(a),(0,o.$Z)(d)}else e.cM(`No ad for element #${d.id}, collapsing`),e.cM(s),(0,o.wo)(d)}}catch(t){e.cM("Exception thrown onSlotRendered"),e.cM(t),this.stalled("e=1")}}stalled(e){(new Image).src=`https://${this.clc_options.h}/stalled.gif?${e}`}defineSlot(t,s){"dfp-isb"===t&&(e.cM("-> targeting - Sidebar: Inline"),s.pubads().setTargeting("Sidebar",["Inline"])),"dfp-tsb"===t&&(e.cM("-> targeting - Sidebar: Right"),s.pubads().setTargeting("Sidebar",["Right"]));const{path:o,sizes:a,zone:i}=(0,n.Z7)(t);e.cM(`Defining slot for ${t}: ${o}, sizes: ${JSON.stringify(a)}`),s.defineSlot(o,a,t).addService(s.pubads())}importGptLibrary(){this.gptImported||(this.gptImported=!0,void 0===this.opt.targeting_consent||this.opt.targeting_consent?(0,o.Gx)("https://securepubads.g.doubleclick.net/tag/js/gpt.js"):(0,o.Gx)("https://pagead2.googlesyndication.com/tag/js/gpt.js"))}importDvLibrary(){this.clc_options.dv_enabled&&(e.cM("Adding DoubleVerify library"),(0,o.Gx)("https://pub.doubleverify.com/dvtag/21569774/DV1289064/pub.js"),e.cM("Adding DoubleVerify onDvtagReady handler"),window.onDvtagReady=function(t,s=750){e.cM("DoubleVerify onDvtagReady called"),window.dvtag=window.dvtag||{},dvtag.cmd=dvtag.cmd||[];const n={callback:t,timeout:s,timestamp:(new Date).getTime()};dvtag.cmd.push(function(){dvtag.queueAdRequest(n)}),setTimeout(function(){const e=n.callback;n.callback=null,e&&e()},s)})}isGptReady(){return"undefined"!=typeof googletag&&!!googletag.apiReady}initGpt(){"undefined"==typeof googletag&&(window.googletag={cmd:(0,o.QZ)(()=>{this.importGptLibrary(),this.importDvLibrary()})})}getUserMeta(){if(this.opt.allowAccountTargetingForThisRequest&&this.clc_options.tgt_e&&this.clc_options.tgt_p>0){if(e.cM("Targeting enabled."),this.clc_options.tgt_p<100){e.cM("Targeting rate limit enabled. Rolling the dice...");const t=Math.floor(100*Math.random())+1;if(e.cM("Rolled "+t+" and the max is "+this.clc_options.tgt_p),t>this.clc_options.tgt_p)return void e.cM("Will not request targeting.")}return e.cM("Will request targeting."),function(e,t,s,n){if(t){const t=new Headers;return t.append("Accept","application/json"),async function(e,t={},s=5e3){if("number"!=typeof s&&null!=s&&!1!==s){if("string"!=typeof s)throw new Error("fetchWithTimeout: timeout must be a number");if(s=parseInt(s),isNaN(s))throw new Error("fetchWithTimeout: timeout must be a number (or string that can be parsed to a number)")}const n=new AbortController,{signal:o}=n,a=fetch(e,{...t,signal:o}),i=setTimeout(()=>n.abort(),s);try{const e=await a;return clearTimeout(i),e}catch(e){throw clearTimeout(i),e}}(s+"?"+new URLSearchParams({omni:e}),{method:"GET",mode:"cors",headers:t},n).then(e=>e.json())}return Promise.reject("No consent")}(this.opt.omni,this.opt.targeting_consent,this.clc_options.tgt_u,this.clc_options.tgt_to).catch(t=>{e.vU("Error fetching user account targeting"),e.vU(t)})}e.cM("Targeting disabled. Will not request account targeting data.")}initDebugPanel(t,s){e.cM("initDebugPanel"),e.cM("Not showing debug panel.")}},window.clcGamLoaderOptions&&(cam.init(),cam.load())})()})();</script> <footer id="footer" class="site-footer js-footer" role="contentinfo"> <div class="site-footer--container"> <nav class="site-footer--nav" aria-label="Footer"> <div class="site-footer--col"> <h5 class="-title"><a href="/">Mathematics Educators</a></h5> <ul class="-list js-primary-footer-links"> <li><a class="js-gps-track -link" data-gps-track="footer.click({ location: 2, link: 2 })" href="/tour">Tour</a></li> <li><a href="/help" class="js-gps-track -link" data-gps-track="footer.click({ location: 2, link: 3 })">Help</a></li> <li><a class="js-gps-track -link" data-gps-track="footer.click({ location: 2, link: 5 })" href="https://chat.stackexchange.com?tab=site&host=matheducators.stackexchange.com">Chat</a></li> <li><a class="js-gps-track -link" data-gps-track="footer.click({ location: 2, link: 13 })" href="/contact">Contact</a></li> <li><a class="js-gps-track -link" data-gps-track="footer.click({ location: 2, link: 14 })" href="https://matheducators.meta.stackexchange.com">Feedback</a></li> </ul> </div> <div class="site-footer--col"> <h5 class="-title"><a class="js-gps-track" data-gps-track="footer.click({ location: 2, link: 1 })" href="https://stackoverflow.co/">Company</a></h5> <ul class="-list"> <li><a href="https://stackoverflow.com" class="js-gps-track -link" data-gps-track="footer.click({ location: 2, link: 15})">Stack Overflow</a></li> <li><a href="https://stackoverflow.co/teams/" class="js-gps-track -link" data-gps-track="footer.click({ location: 2, link: 29 })">Teams</a></li> <li><a href="https://stackoverflow.co/advertising/" class="js-gps-track -link" data-gps-track="footer.click({ location: 2, link: 21 })">Advertising</a></li> <li><a href="https://stackoverflow.co/advertising/employer-branding/" class="js-gps-track -link" data-gps-track="footer.click({ location: 2, link: 20 })">Talent</a></li> <li><a class="js-gps-track -link" data-gps-track="footer.click({ location: 2, link: 1 })" href="https://stackoverflow.co/">About</a></li> <li><a class="js-gps-track -link" data-gps-track="footer.click({ location: 2, link: 27 })" href="https://stackoverflow.co/company/press/">Press</a></li> <li><a class="js-gps-track -link" data-gps-track="footer.click({ location: 2, link: 7 })" href="https://stackoverflow.com/legal">Legal</a></li> <li><a class="js-gps-track -link" data-gps-track="footer.click({ location: 2, link: 8 })" href="https://stackoverflow.com/legal/privacy-policy">Privacy Policy</a></li> <li><a class="js-gps-track -link" data-gps-track="footer.click({ location: 2, link: 37 })" href="https://stackoverflow.com/legal/terms-of-service/public">Terms of Service</a></li> <li id="consent-footer-link"><button type="button" data-controller="cookie-settings" data-action="click->cookie-settings#toggle" class="s-btn s-btn__link py4 js-gps-track -link" data-gps-track="footer.click({ location: 2, link: 38 })" data-consent-popup-loader="footer">Cookie Settings</button></li> <li><a class="js-gps-track -link" data-gps-track="footer.click({ location: 2, link: 39 })" href="https://stackoverflow.com/legal/cookie-policy">Cookie Policy</a></li> </ul> </div> <div class="site-footer--col site-footer--categories-nav"> <div> <h5 class="-title"><a href="https://stackexchange.com" data-gps-track="footer.click({ location: 2, link: 30 })">Stack Exchange Network</a></h5> <ul class="-list"> <li> <a href="https://stackexchange.com/sites#technology" class="-link js-gps-track" data-gps-track="footer.click({ location: 2, link: 24 })"> Technology </a> </li> <li> <a href="https://stackexchange.com/sites#culturerecreation" class="-link js-gps-track" data-gps-track="footer.click({ location: 2, link: 24 })"> Culture & recreation </a> </li> <li> <a href="https://stackexchange.com/sites#lifearts" class="-link js-gps-track" data-gps-track="footer.click({ location: 2, link: 24 })"> Life & arts </a> </li> <li> <a href="https://stackexchange.com/sites#science" class="-link js-gps-track" data-gps-track="footer.click({ location: 2, link: 24 })"> Science </a> </li> <li> <a href="https://stackexchange.com/sites#professional" class="-link js-gps-track" data-gps-track="footer.click({ location: 2, link: 24 })"> Professional </a> </li> <li> <a href="https://stackexchange.com/sites#business" class="-link js-gps-track" data-gps-track="footer.click({ location: 2, link: 24 })"> Business </a> </li> <li class="mt16 md:mt0"> <a href="https://api.stackexchange.com/" class="-link js-gps-track" data-gps-track="footer.click({ location: 2, link: 24 })"> API </a> </li> <li> <a href="https://data.stackexchange.com/" class="-link js-gps-track" data-gps-track="footer.click({ location: 2, link: 24 })"> Data </a> </li> </ul> </div> </div> </nav> <div class="site-footer--copyright fs-fine md:mt24"> <ul class="-list -social md:mb8"> <li><a class="js-gps-track -link" data-gps-track="footer.click({ location: 2, link:4 })" href="https://stackoverflow.blog?blb=1">Blog</a></li> <li><a href="https://www.facebook.com/officialstackoverflow/" class="-link js-gps-track" data-gps-track="footer.click({ location: 2, link: 31 })">Facebook</a></li> <li><a href="https://twitter.com/stackoverflow" class="-link js-gps-track" data-gps-track="footer.click({ location: 2, link: 32 })">Twitter</a></li> <li><a href="https://linkedin.com/company/stack-overflow" class="-link js-gps-track" data-gps-track="footer.click({ location: 2, link: 33 })">LinkedIn</a></li> <li><a href="https://www.instagram.com/thestackoverflow" class="-link js-gps-track" data-gps-track="footer.click({ location: 2, link: 36 })">Instagram</a></li> </ul> <p class="md:mb0"> <span>Site design / logo © 2025 Stack Exchange Inc; </span> <span>user contributions licensed under </span> <a class="-link s-link td-underline" href="https://stackoverflow.com/help/licensing">CC BY-SA</a> <span>. </span> <span id="svnrev">rev 2024.12.20.20747</span> </p> </div> </div> </footer> <!-- Google tag (gtag.js) --> <script async src="https://www.googletagmanager.com/gtag/js?id=G-S812YQPLT2"></script> <script> window.dataLayer = window.dataLayer || []; function gtag() { dataLayer.push(arguments); } </script> <script> StackExchange.ready(function() { var ga3Settings = { autoLink: ["stackoverflow.blog","info.stackoverflowsolutions.com","stackoverflowsolutions.com"], sendTitles: true, tracker: window.ga, trackingCodes: [ 'UA-108242619-5' ], checkDimension: 'dimension42' }; var customGA4Dimensions = {}; customGA4Dimensions["requestid"] = "52b539a6-79e8-4add-a499-9ac156a8f381"; customGA4Dimensions["routename"] = "Questions/Show"; customGA4Dimensions["post_id"] = "8456"; customGA4Dimensions["tags"] = "|undergraduate-education|self-learning|lecture-notes|"; var ga4Settings = { tracker: gtag, trackingCodes: [ 'G-S812YQPLT2' ], consentsToPerformanceCookies: "denied", consentsToTargetingCookies: "denied", eventParameters: customGA4Dimensions, checkForAdBlock: true, sendTitles: true, trackClicks: false, }; StackExchange.ga.init({ GA3: ga3Settings, GA4: ga4Settings }); StackExchange.ga.setDimension('dimension2', '|undergraduate-education|self-learning|lecture-notes|'); StackExchange.ga.setDimension('dimension3', 'Questions/Show'); StackExchange.ga.setDimension('dimension7', "1735780817.210475740"); StackExchange.ga.trackPageView(); }); </script> <script src="https://cdn.cookielaw.org/scripttemplates/otSDKStub.js" charset="UTF-8" data-document-language="true" data-domain-script="cb0f3c87-b769-4e66-bbaa-377f9194216d"></script> <script defer src="https://cdn.sstatic.net/Js/modules/cookie-consent.en.js?v=36bebc18e04f"></script> <script>(function(){function c(){var b=a.contentDocument||a.contentWindow.document;if(b){var d=b.createElement('script');d.innerHTML="window.__CF$cv$params={r:'8fb6ecf968d444b3',t:'MTczNTc4MDgxNy4wMDAwMDA='};var a=document.createElement('script');a.nonce='';a.src='/cdn-cgi/challenge-platform/scripts/jsd/main.js';document.getElementsByTagName('head')[0].appendChild(a);";b.getElementsByTagName('head')[0].appendChild(d)}}if(document.body){var a=document.createElement('iframe');a.height=1;a.width=1;a.style.position='absolute';a.style.top=0;a.style.left=0;a.style.border='none';a.style.visibility='hidden';document.body.appendChild(a);if('loading'!==document.readyState)c();else if(window.addEventListener)document.addEventListener('DOMContentLoaded',c);else{var e=document.onreadystatechange||function(){};document.onreadystatechange=function(b){e(b);'loading'!==document.readyState&&(document.onreadystatechange=e,c())}}}})();</script></body> </html>